Mental Health Final

Réussis tes devoirs et examens dès maintenant avec Quizwiz!

Chemical Restraint

2 mg ativan/ 5 mg Haldol

In the patient in Question 39.3, urine toxicology confirms intoxication with amphetamines. Which of the following withdrawal syndromes would be expected? A) Diarrhea, piloerection, yawning B) Delirium, autonomic hyperactivity, visual or tactile hallucinations C) "Crash" of mood into depression, lethargy, increased appetite D) Tremor, headache, hypertension E) Postural hypotension, psychomotor agitation, insomnia

"Crash" of mood into depression, lethargy, increased appetite

A 24 heard old man presents to a therapist. Which of the following statements made by the patient is most consistent with avoidant personality disorder? A. "I have a couple of close friends, but it is very hard to make friends. I'm afraid most people wouldn't want me around." B. "I'm usually fine around people. It's just when I'm around a lot of people I've never met before that I get freaked out." C. "I'm afraid that people are plotting against me." D. "My mom thinks that I have a problem with people. I can take them or leave them." E. "My girlfriend thinks I have a problem with people, like with her friends. What do you think?"

"I have a couple of close friends, but it is very hard to make friends. I'm afraid most people wouldn't want me around."

A 22 year old woman is referred to a psychiatrist by her mother because she has few friends and "doesn't socialize like other people her age." Which of the following statements is most consistent with a diagnosis of schizotypal personality disorder? A. "I have plenty of friends. Every time I make new friends, they just betray me. People are always working behind my back to sabotage me. I don't need that hassle." B. "I don't need friends. My life is fine the way it is." C. "I would love to have new friends, but no one will want to be my friends once they get to know me." D. "I have plenty of friends. The stars have told me I should have two friends, and that's exactly what I have." E. "I would love to have new friends. I'm only whole when I'm with my best friend, and the more time I spend with my friend the less likely I am to be alone."

"I have plenty of friends. The stars have told me I should have two friends, and that's exactly what I have."

A 22 year old single graduate student with narcissistic personality disorder is admitted to a hospital after a car accident in which his right femur is fractured. A medical student has been assigned to follow the patient, but when she enters the room and introduces herself as a medical student, the patient states, "Oh, I wouldn't let a medical student touch me -- I need someone with much more experience than you." Which of the following statements by the medical provider is most likely to lead to a successful interview with this patient? A. "I know this will be boring for you, but it's just one of the things that you will have to put up with in the hospital." B. "I know you must be scared to be in the hospital, but you will be safe here." C."I'm told that you are a very articulate person, so I'm hoping you'll teach me what I need to know." D. "I understand that you think you deserve only the best, but I have been assigned to you." E. "Please don't make this difficult; I have to interview you as part of my job."

"I'm told that you are a very articulate person, so I'm hoping you'll teach me what I need to know."

A 36-year-old woman comes into your office for a new patient evaluation. When reviewing her past medical history, she tells you she has a diagnosis of schizoaffective disorder. Which of the following statements is most consistent with a diagnosis of schizoaffective disorder? A "I had a depressive episode last year following my divorce. During that time, I heard the devil talking to me and telling me I was worthless." B "I have chronic emptiness with daily mood swings and hear voices inside my head." C "I have had several psychotic episodes throughout my life. When I'm not hearing voices and focused on my delusions, I spend most of my time alone and I'm not really interested in much." D "I've had several psychotic episodes in my life. In between, I sometimes function okay, but I have had a couple times where my mood got really low and I crawled under the covers for weeks." E "I've had several episodes where I go days without sleep because I don't need any sleep. During those times, I hear God talking to me and telling me all the work He wants me to do to cure cancer and solve world hunger. In between these times, I'm a successful CEO for a financial management company."

"I've had several psychotic episodes in my life. In between, I sometimes function okay, but I have had a couple times where my mood got really low and I crawled under the covers for weeks."

An epidemiological survey of the general public showed between 9% and 16% met criteria for one or more personality disorders. In psychiatric samples the prevalence is even higher. What percentage of personality disorders would you expect to find in an outpatient psychiatric sample? A. 15 to 30% B. 30 to 50% C. 50 to 70% D. 60 to 80% E. 70 to 90%

30 to 50%

A 54-year-old man is admitted to the hospital for elective surgery. He has been through alcohol rehabilitation but has continued to struggle with his drinking. He alerts the primary service taking care of him that he has continued to drink up to the time of his admission. In what time frame after cessation of all drinking is he at most risk for DTs? A) 6 to 8 hours B) 8 to 12 hours C) 12 to 24 hours D) 48 to 96 hours E) Over 1 week

48 to 96 hours

The woman in Questions 13.2 and 13.3 is seen 1 year later for a return visit. She has remained on the citalopram at the same dose, and she is tolerating it well, but she worries about "always having to take medication," and requests it to be discontinued. What is her lifetime risk of reoccurrence if not maintained on medication? A 10% B 25% C 50% D 85% E 100%

85%

Which of the following scenarios is most consistent with antisocial personality traits? A. A 13-year-old boy living in an extremely economically depressed area joins a street gang to avoid being beaten by competing gangs B. A 38-year-old man with a drug addiction is arrested seven times for retail theft C. A 67-year-old man, who is a CEO, embezzles from his company, is unfaithful to his third wife, and has been involved in covering up corporate malfeasance from federal investigators D. A 42-year-old homeless woman with schizophrenia is arrested on misdemeanor trespassing charges five times in the last 2 years E. A 28-year-old woman has a history of prostitution, drug abuse, and multiple suicide attempts with over 20 inpatient psychiatric admissions

A 67-year-old man, who is a CEO, embezzles from his company, is unfaithful to his third wife, and has been involved in covering up corporate malfeasance from federal investigators

Which of the following statements is accurate about the presentation of narcissistic personality disorder? A. Eye contact is typically avoided B. The patient is frequently disoriented to person, place, and time C. NPD is typically diagnosed alone, without known comorbid conditions D. A depressed mood due to dysthymia or major depressive disorder may be noted

A depressed mood due to dysthymia or major depressive disorder may be noted

A 48 year old woman presents to a psychotherapist. The patient lives a very secluded life, largely consumed by working nights as a janitor at a department store and taking care of her elderly mother. She complains of feeling lonely and is aware that she has a great deal of difficulty relating to other people. Which of the following conditions would most distinguish her issues from a person with schizoid personality disorder? A. Family history of a cousin with schizophrenia B. A desire to engage in interpersonal relationships C. Lack of hallucinations or delusional thinking D. A history of alcoholism

A desire to engage in interpersonal relationships

Which of the following statements is accurate about the workup of narcissistic personality disorder? A. A toxicology screen is generally recommended B. Specifically personality tests are required for definitive diagnosis C. Diagnosis requires obtaining a history from the patient's family and friends D. Cognitive function testing is strongly encouraged

A toxicology screen is generally recommended

A 20-year-old man is brought to the emergency department (ED) by his friends in a scared and agitated state. He had been playing basketball several hours ago with his friends when he suddenly began complaining of being stung by bees. He began behaving erratically and saying that bees were coming out of his ears and mouth. He began accusing one of his friends of plotting against him and blaming him for his condition. He became agitated and ran away from the basketball court, darting in and out of traffic on the adjoining street. His friends had to track him down and force him to come to the hospital. His friends report that some of them had been smoking "weed" prior to the game, and the patient was a "newbie to weed." His friends do not believe he has had any psychiatric problems in the past but cannot be sure. On examination, the patient appears anxious, apprehensive, and fearful. He looks around the room furtively, repeatedly spitting into a cup. He is fully aware of his surroundings, but his attention span is very short. His memory is intact and fully oriented. He no longer suspects his friend and holds out the cup he has been spitting into for the doctor to inspect for bees. He finds it unbelievable that he has bees crawling out of himself but cannot help feeling the sensation. Physical examination reveals redness of eyes and an elevated pulse rate. Otherwise, the physical examination is normal. Neurologic examination is within normal limits. The patient complains of the bees "sucking out all [his] energy making [him] very hungry." A) What is the most likely diagnosis? B) What is the best diagnostic test?

A) Cannabis intoxication B) Urine toxicology screening

Ms. K, a 60-year-old woman, is brought by her sister to see a psychiatrist at a local mental health center with complaints that Ms. K has "changed" over the past year. According to the sister, Ms. K appears anxious all the time and stays by herself in her room, not socializing with anyone. She even quit her "very good" job at the local school. Ms. K has not been eating well and has lost significant weight within the time. She used to be a "fun-loving" person before. The sister wants the physician to "bring her sister back."Ms. K denies that she is sad. She worries about her health because she is convinced that mercury poisoning has caused all her symptoms, but the doctors "do not seem to believe her." She reports that about a year ago, she visited her dentist and was informed that her dental filling had fallen off. That night, her husband jokingly said that she may have mercury poisoning since "silver dental fillings may cause it." Ms. K then began to research mercury poisoning on the Internet. Within several months, she developed all the symptoms of mercury poisoning that were mentioned on various websites. She started to isolate herself in her room to further look up the symptoms on the Internet. She often gets into arguments with her husband because he thinks that "she is getting paranoid." She gets irritable when her sisters visit and refuses to go out to eat with them. Ms. K cannot sleep well because thinking about her illness keeps her awake at night. She reports she cannot eat much.After 6 months of symptoms at home, her family took her to see her primary care provider (PCP), who performed a thorough physical and neurologic exam and ran several tests, including blood and urine tests, to measure mercury levels. All of these tests were within normal limits. Ms. K seemed relieved for few days but the Internet searches began again, and so did all the symptoms. She was taken back to her PCP, who then referred her to the psychiatrist. There is no personal or family history of major psychiatric illnesses or substance abuse. Psychiatric review of systems reveals mild to moderate anxiety, which has never been treated with psychotropic medications. Mental status exam reveals no paranoid thoughts or hallucinations. Ms. K has a calm demeanor with no agitation and very rational thinking. She states, "I know I may be having anxiety for no reason, but I am still having all the symptoms of mercury poisoning." A) What is the most likely diagnosis for this patient? B) What is the next step in treating this patient?

A) Illness anxiety disorder B) Patients with illness anxiety disorder may or may not be willing to receive care from a mental health provider, in which case a PCP may need to continue care with such patients. Either way, first work on building a solid rapport with the patient. Providing supportive psychotherapy, validating the patient's symptoms without reinforcing illness behavior, and providing psychoeducation is important. It should be understood that this disorder may be perpetuated by the patient's use of the sick role as a solution to her problems. Invasive diagnostic techniques or procedures should be avoided unless there is convincing objective evidence that they are necessary. Group therapy, behavioral therapy, insight-oriented therapy, and hypnosis may help. Although there is no long-term benefit, pharmacotherapy to target anxiety might help.

A 19-year-old man presents to a psychiatrist insisting, "I have schizophrenia and need to be admitted." He describes hearing voices telling him to kill himself for the past several days. He says that he is possessed by the devil. The patient denies feeling depressed but insists he will hurt himself if he is not admitted to a hospital immediately. He has vague suicidal plans and says he will find some ways to kill himself but cannot be more specific. He has no prior history of psychiatric treatment or complaints, no medical problems, and is not taking any medication. He drinks one or two beers a week and denies using drugs. At the end of the interview, he again requests hospitalization. He then adds that he is currently on leave from the Navy and is due back on his ship, which is leaving in 2 days.On a mental status examination, the patient is initially cooperative and forthcoming but becomes increasingly irritated when asked to give more details about his symptoms. He has good hygiene and maintains good eye contact. His mood and affect are euthymic and full range. His thought processes are logical, without looseness of association or thought blocking. His thought content is notable for suicidal ideation but no homicidal ideation. He reports having delusions and auditory hallucinations but doesn't seem to be responding to any internal stimuli. His insight seems good considering the severity of his symptoms. A) What is the most likely diagnosis? B) How would you approach this patient?

A) Malingering B) Obtain collateral information (if possible) from family and/or friends. Approach the patient in a nonthreatening manner, remaining neutral and avoiding abrupt confrontations or accusations of lying. Be aware of implicit countertransference reactions. Explore and validate the patient's feelings regarding his military duty. Discuss that findings indicate no medical basis for the patient's symptoms, and therefore no diagnosis can be made. Refer him for an appropriate follow-up (if possible).

What behavior would be most typical for a patient diagnosed with schizoid personality disorder? A. Chooses solitary activities B. Odd thinking and speech C. Reckless disregard for safety of others D. Uses physical appearance to draw attention

Chooses solitary activities

A dissociative disorder characterized by experiences of unreality or being detached from one's thoughts, feelings, sensations, or actions.i.e. feeling like an outside observer

Depersonalization

The patient in Question 55.1 (narcissistic, medical student) would be most likely to become depressed after which of the following life occurrences? A. Aging B. Graduation C. Job change D. Marriage E. Moving to a new city

Aging

Which of the following is not recognized as one of the diagnostic criteria of major depressive disorder according to the Diagnostic and Statistical Manual of Mental Disorders, fifth edition (DSM-5)? A. Fatigue B. Agitation not influenced by external stimuli C. Sleep disturbance D. Significant weight change

Agitation not influenced by external stimuli

A 55-year-old unresponsive woman is brought to the emergency department after an apparent suicide attempt. Earlier that day, she refilled her monthly prescription for lorazepam, which she had been prescribed for panic disorder. The empty pill bottle was found on her nightstand by the paramedics. Concurrent ingestion of which of the following substances is most likely to worsen the prognosis of her overdose? A) Cannabis B) Cocaine C) Alcohol D) Lysergic acid diethylamide (LSD)

Alcohol

A 45-year-old married man presents to his primary care provider with a chief complaint of fatigue lasting for the past 9 months. He states that he goes to sleep easily enough but then wakes up repeatedly throughout the night. He has had this problem since he was injured on the job 9 months ago. On further questioning, he reports a low mood. He states that his alcohol consumption is 6 to 12 beers a day, as well as several ounces of hard liquor to "take the edge off the pain." He discloses that it takes more alcohol than it used to in order to "get relaxed." The patient states that he has experienced several blackouts caused by drinking during the past 2 months and admits that he often has a drink of alcohol first thing in the morning to keep him from feeling shaky. Despite receiving several reprimands at work for tardiness and poor performance and his wife threatening to leave him, he has been unable to stop drinking. On his mental status examination, the patient is alert and oriented to person, place, and time. He appears rather haggard, but his hygiene is good. His speech is of normal rate and tone, and he is cooperative with the practitioner. His mood is noted to be depressed, and his affect is congruent, although full range. Otherwise, no abnormalities are noted. dx?

Alcohol use disorder

Which of the following findings is a negative symptom of schizophrenia? A Alogia B Auditory hallucinations C Delusions D Disorganized speech

Alogia

A 28-year-old man is brought into the emergency center for an overdose of prescribed medications, which includes benzodiazepines. He also admits to being habituated to his prescribed medication. After stabilization for the acute problems, he is subsequently transferred to another facility for detoxification. Which of the following benzodiazepines is most likely to cause withdrawal syndrome? A) Chlordiazepoxide B) Clonazepam C) Alprazolam D) Diazepam E) Lorazepam

Alprazolam

Which of the following is a recognized criterion for narcissistic personality disorder, according to the DSM-5? A. Interpersonally explosive behavior B. A high degree of empathy that is willingly rejected C. An absence of empathy toward others D. None of the above

An absence of empathy toward others

A 16-year-old girl has been admitted with a 3-week history of sudden irritability, impulsive buying, and disappearing at night with older men. Her need for sleep is decreased; she has flight of ideas and grandiose thoughts about being an advisor to a presidential candidate. Test results indicate she is pregnant. Which of the following is the most appropriate statement that should be made to her parents? A Treatment with an SSRI antidepressant is a reasonable alternative to mood stabilizers in a pregnant patient B Given the fact that she is pregnant, she should be kept secluded on an inpatient unit during the first trimester of pregnancy with no medications C An atypical antipsychotic may be the best choice for managing both psychotic features and mood disturbance associated with her bipolar disorder, especially during the first trimester of pregnancy D Psychotherapy will have little role in the treatment of her bipolar disorder E Lithium, divalproex, and carbamazepine are all reasonable first-choice mood stabilizers for this patient

An atypical antipsychotic may be the best choice for managing both psychotic features and mood disturbance associated with her bipolar disorder, especially during the first trimester of pregnancy

A 36 year old man with narcissistic personality disorder calls your office asking for an appointment with the "best therapist in the clinic." One of his complaints is difficulties in his relationships with colleagues. The patient states, "They are not giving me the credit I deserve for my accomplishments at the law firm." What is the most likely reason the patient is seeking treatment? A. Anger B. Anxiety C. Attempting to identify with others D. Grandiose thinking E. Seeking medication

Anger

Which of the following side effects common to SSRIs is the woman in Question 13.2 (citalopram 6 weeks) most likely to complain of in the future? A Anorgasmia B Headaches C Insomnia D Nausea E Tremor

Anorgasmia

A 17-year-old honors student is brought to the emergency department by his parents. In the last academic quarter, his grades have suddenly dropped, he is irritable with friends and family, has no energy, does not go to bed until 1 AM, and has a poor appetite. He also has auditory hallucinations in which a man's voice tells him that he is a "lazy bastard" and that his family "would be better off with him dead." Which of the following would be the most appropriate initial pharmacologic treatment plan? A Benzodiazepine B Antidepressant and antipsychotic medications C Antidepressant medication D Antipsychotic medication and a benzodiazepine E Antidepressant medication and lithium

Antidepressant and antipsychotic medications

A 20-year-old musician comes to his primary care provider's office because he has been feeling depressed, fatigued, and unmotivated for the last few weeks. He wants help getting his "muse" back. Physical examination and laboratory studies are unremarkable. He is prescribed fluoxetine with a plan to follow up in a month. The patient's girlfriend brings him back to clinic 2 weeks later, concerned that he is "acting different." She reports that the patient plays guitar all day and late into the night. She complains that he has spent all their money on new guitars and keyboards. She does not want to "crush his dreams, but he's reaching a bit high trying to book tours with the Rolling Stones." The patient reports feeling "Better than ever!" He talks quickly and informs the clinician that he has to get going so he can keep practicing for "the big gig." What is the most likely diagnosis? A Antidepressant-induced bipolar disorder B Bipolar disorder due to another medical condition C Major depressive disorder D Narcissistic personality disorder E Schizoaffective disorder

Antidepressant-induced bipolar disorder

What is the best treatment option for the patient in 56.1 (paranoid)? A. Antianxiety medication B. Antipsychotic medication C. Hospitalization D. Psychotherapy E. Reassure the patient that he is safe

Antipsychotic medication

Patient presents as a 26 year old male with an extensive criminal record who assaults an elderly man in an attempt to rob his bicycle. During the interview, he is polite and has a normal affect and describes with excitement how he hit the old man with a wooden baseball bat prior to robbing him

Antisocial personality disorder

A 10-year-old girl is brought in for treatment by her father because the child is not sleeping well, has lost 7 lb because of a decreased appetite, seems to be tired much of the time, is irritable and tearful most of the time, refuses to go to school or play with her friends, and has been spending most of her time locked in her room. These symptoms gradually began a few months after the sudden death of the girl's mother due to suicide. The father is worried about suicide risk in the girl but does not want to ask her about it, as he fears putting suicidal ideas in her mind. Which is the best way to address suicide risk in this patient? A Avoid bringing up issue of suicide with the patient. B Ask the child about suicidal thoughts. C Ask the father to keep a close eye on the patient for any developing signs of suicidal ideation D A 10-year-old is too young to attempt suicide. E Involve a friend or family member to get collateral history

Ask the child about suicidal thoughts

A patient with obsessive-compulsive personality disorder may also be categorized in a grouping of disorders named Cluster C personality disorders. Which other personality disorder is part of Cluster C? A. Antisocial personality disorder B. Schizotypal personality disorder C. Narcissistic personality disorder D. Avoidant personality disorder E. Borderline personality disorder

Avoidant personality disorder

Patient presents as a 47 year old female office manager who refuses to attend the annual holiday party because she is afraid her coworkers will not like her. She would like to meet people but feels too shy to initiate a conversation

Avoidant personality disorder

A 30-year-old woman with an 18-year history of daily intravenous heroin use comes into the community rehabilitation clinic asking to be "detoxed" in order to "kick my heroin habit" for good. Though she is motivated for treatment, she currently is shivering, with rhinorrhea, tearing, gagging, and grabbing her abdomen in pain due to cramps. When questioned about her last use, she states that shortly before arriving, a peer gave her "something different to shoot up." Which of the following substances would be most likely to cause her current symptoms? A) Alprazolam B) Buprenorphine C) Cocaine D) Methadone E) Morphine

Buprenorphine

An otherwise healthy 36 year old man is being admitted for detoxification owing to alcohol abuse. He has a history of drinking 8-10 beers and about 5 shots of scotch nightly for at least the last 5 years. Which is the best medication to help control withdrawal symptoms in the first 48-72 hours?

Benzodiazepine

What is the treatment for akathisia (motor restlessness with a compelling urge to move and inability to sit still)?

Benzodiazepines, such as lorazepam

A 22-year-old woman presents to your clinic with concerns about her mental health. She says that last month she felt very depressed, was sleeping 12 hours per night, and missed several days of work. Last week, she suddenly felt significantly more energetic, was sleeping three to four hours per night, and was gambling excessively. During your interview, she talks very quickly and is easily distracted. She says that her family has expressed concern because she told them last night that she was having thoughts of suicide. Which of the following is the most likely diagnosis? A Attention deficit hyperactivity disorder B Bipolar disorder C Borderline personality disorder D Schizoaffective disorder

Bipolar disorder

A 33-year-old writer is brought to the emergency department (ED) by her sister, who voices concern that her sibling is acting "out of control." The patient laughs at her sister's accusation and rapidly retorts, "I feel great! She's the one with something wrong." The patient paces around the room, speaking rapidly. The ED provider attempts to redirect the interview several times, but the patient keeps talking. Her sister reports that the patient was like this several months ago, but otherwise has been normal. She remembers that both episodes seemed to occur around the time of her sister's period. The patient responds by chanting, "Yes! Yes! I've got the PMS!" The patient has no known medical problems, substance use, or family history of psychiatric illness. What is the most likely diagnosis? A Bipolar disorder B PMDD C PMS D Schizophrenia E Substance-induced bipolar disorder

Bipolar disorder

A 25-year-old man presents to the emergency department after being arrested for stealing from a department store. He states that he is the president of the United States. He also says that he has not slept for three days and does not feel like he needs to sleep. During the interview, he speaks rapidly and is easily distracted. He denies any use of substances, and the toxicology screen is negative. Which of the following is the most likely diagnosis? A Bipolar disorder type 1 B Bipolar disorder type 2 C Cyclothymic disorder D Major depressive disorder

Bipolar disorder type 1

A 35-year-old woman presents with anhedonia and depressed mood for two weeks. In the past four days, she has been easily distractible and irritable, but her occupation is not affected. Which of the following is the most likely diagnosis? A Bipolar disorder type II B Cyclothymic disorder C Dysthymic disorder D Major depressive disorder

Bipolar disorder type II

Patient presents as a 25 year old female doctoral student comes to your office for a checkup. When you enter the room she is visibly upset and states that all of the office staff are incompetent and that you're the only medical provider who has ever understood her despite only having one brief clinical encounter. When you tell her that you cannot prescribe her the pain medication she requested, she states that you are incompetent and abruptly leaves the appointment

Borderline personality disorder

Which of the following is the most appropriate management of acute psychosis in a patient with schizophrenia? A Amitriptyline (Elavil) B Fluphenazine (Prolixin) C Lithium (Eskalith) D Sertraline (Zoloft)

Fluphenazine (Prolixin)

A 30-year-old man presents to his primary care provider's office complaining of being "down" for the last month. He has been suffering from difficulty falling and staying asleep, severe fatigue, guilt, poor appetite, and thoughts of wanting to take his life. He does not ever recall feeling this bad. He has stopped talking to his friends and has no interest in doing anything. He is diagnosed with major depressive disorder. He is open to treatment but has heard that antidepressants cause sexual dysfunction. His concerns are noted, and he is started on an antidepressant medication. He presents to the emergency department 3 days later after an episode of generalized tonic-clonic seizures. Which of the following antidepressants was he most likely prescribed? A Sertraline B Fluoxetine C Imipramine D Bupropion E Phenelzine

Bupropion

A 20-year-old college student presents to the ED with acute agitation and psychosis. He has been smoking marijuana since age 16 years, and his use has increased to the point where his roommates note that all he does is "smoke pot." He began distancing himself from others in the dorm over the last year, complaining that they were trying to kill him by poisoning his food. His school grades and self-care have gone downhill considerably. However, he does not feel his cannabis use is problematic and states that "weed is the only thing that has kept me sane!" What is the most likely diagnosis in this patient?

Cannabis-induced psychosis

30-year-old woman with a known history of anxiety presents to her provider to have her anxiety medications refilled. She has been using more of her medications recently because she has had some new symptoms. In particular, she is experiencing intrusive thoughts and feels compelled to do things to quiet the thoughts, which sometimes drives her to take her anxiety medicine. These unwanted thoughts are starting to interfere with her ability to get to work on time and focus on work while she is there. Which of the following would be most consistent with the suspected diagnosis? a. Avoidance of public transportation or crowds due to fear of an inability to escape b. Checking to make sure all appliances and lights are off before leaving a room due to concern for fire c. Checking weight repeatedly and adjusting diet due to fear of gaining weight d. Impulse to pick skin and repeated attempts to stop the habit

Checking to make sure all appliances and lights are off before leaving a room due to concern for fire

Which of the following physical complications would be most likely to occur in the patient in Question 39.1? (A 50-year-old homeless man is brought to the emergency department by the police for disruptive behavior. On mental status examination, he has an elevated affect, but he also has psychomotor agitation and paranoia; he says he "feels fantastic" but is wary of answering any questions, quickly becoming irritated. On physical examination, the patient exhibits a moderately elevated blood pressure and pulse rate. He is most likely intoxicated with which of the following substances?) A) Chest pain B) Delirium C) Hypothermia D) Respiratory depression

Chest pain

Which of the following is indicated for treatment of obsessive-compulsive disorder? a. Amitriptyline b. Clomipramine c. Doxepin d. Imipramine

Clomipramine

A 25-year-old man arrives at the emergency department with gastrointestinal distress, muscle aches, rhinorrhea, lacrimation, and an anxious mood. He had become addicted to prescription opioids and now buys pills on the street. He says he works as an accountant and just "wants to kick this thing once and for all." Which of the following medications would be most helpful in ameliorating his symptoms? A) Disulfiram B) Clonidine C) Haloperidol D) Lorazepam E) Naloxone

Clonidine

A 25-year-old man with refractory schizophrenia who is being treated with multiple medications has developed left-sided chest pain. Laboratory studies show decreased white blood cell count, decreased absolute neutrophil count, and elevated troponin. Which of the following medications is he taking? A Clozapine B Olanzapine C Risperidone D Ziprasidone

Clozapine

A 45-year-old man with a history of schizophrenia and alcohol use disorder is brought in by an ambulance after he was found sleeping on the floor of a local homeless shelter. He appears drowsy but arousable, and he mumbles, "The voices are killing me." He admits to taking a bottle of lorazepam because "I just couldn't take it anymore." Which of the following antipsychotics has been associated with decreased suicide attempts? A Haloperidol B Clozapine C Quetiapine D Lurasidone E Paliperidone

Clozapine

A 46 year old man is brought to the ED at 3am by his wife because of his constant drinking. Which of the below vitamins should be replaced first in this patient?

Cobalamin (vitamin B12)

A 30 year old man presents in a state of euphoria, but he is also demonstrating impairment of judgement. He was walking on the railing of a local bridge before being brought to the ED by his friends. His friends relay that he has been skipping work or arriving late over the past few weeks. His pupils are dilated, and he has tract marks to the left arm. His BP is elevated, and he is diaphoretic. His clothes are too loose. The manner in which he relates to you is somewhat condescending and he is grandiose in his thinking. There is no history of psychiatric illness, and no medical illness is present. What is the most likely diagnosis?

Cocaine

A 50-year-old homeless man is brought to the emergency department by the police for disruptive behavior. On mental status examination, he has an elevated affect, but he also has psychomotor agitation and paranoia; he says he "feels fantastic" but is wary of answering any questions, quickly becoming irritated. On physical examination, the patient exhibits a moderately elevated blood pressure and pulse rate. He is most likely intoxicated with which of the following substances? A) Alcohol B) Barbiturates C) Benzodiazepines D) Cocaine E) Opiates

Cocaine

Girl feeling like she has ants all over her body and gets dropped off by 2 men who drive away what is she on?

Cocaine

Which of the following features must be present in a patient's history for schizotypal personality disorder to be diagnosed? A. Auditory hallucinations B. Cognitive and perceptual distortions C. Impulsive or manipulative behaviors D. Paranoid ideations E. Unstable and intense relationships

Cognitive and perceptual distortions

A 39-year-old man is evaluated by mental health services in prison. He has a history of multiple arrests as both an adult and a juvenile. After several interviews, a diagnosis of antisocial personality disorders is confirmed. He has a history of multiple psychiatric hospitalizations after suicide attempts and was in special education programming as a child. Which psychiatric diagnosis is most likely to have occurred comorbidly in such an individual? A. Attention deficit/hyperactivity disorder (ADHD) B. Cocaine dependence C. Traumatic brain injury D. Major depression E. Conduct disorder

Conduct disorder

A 60-year-old man is brought to the emergency department by his wife for "confusion." She reluctantly confides to the staff that he is a "heavy drinker" and that he has drunk up to a case of beer almost every day for the past 30 years. Although he has not changed his alcohol intake significantly, over the past year he has eaten less, preferring alcohol to large meals. She has noticed a gradual weight loss as a result. His last drink was earlier that day. Which of the following would be the most likely finding on the mental status examination of this patient? A) Confabulation B) Delusions C) Elevated affect D) Fluctuating consciousness E) Loose associations

Confabulation

A 20-year-old female with dizziness, along with numbness and severe weakness on the left side of her body and her left leg. She also complains of moderate low back pain. The patient was recently involved in a bicycle collision with a motor vehicle 3 days ago where she was clipped while crossing an intersection, causing her to crash hard on her left side. She doesn't remember if she hit her head, but she was wearing a helmet. She was taken to the local, rural hospital to screen for a concussion, which came back negative. She had a CT of the head in the ED which was normal and she was released quickly from the hospital. Since the injury 3 days ago, the patient reports her dizziness has converted to double vision, has difficulty swallowing like there is a lump in her throat, and notices occasional slurred words. She continues to have difficulty walking and loss of balance since the accident. She reports having increased stress and difficulty completing school work in the past couple of weeks. Her physical exam and vital signs are all normal. Her neurological exam reveals inconsistent paresthesia in dermatomes of the LLE.

Conversion disorder (functional neurological symptom disorder) / treatment = therapy +/- short-term anxiolytics

A 30 year old man comes into your clinic after being referred by his primary care provider. He lives with his mother and relies on her to make everyday decisions for him. He has never worked and depends on her for financial support. He lacks self-confidence and is uncomfortable when left alone. Since his mother's diagnosis of cancer, the patient is preoccupied with the fear of his mother dying and being left alone to care for himself. You believe he is suffering from a personality disorder. Which is the most likely disorder? A. Avoidant personality disorder B. Borderline personality disorder C. Dependent personality disorder D. Histrionic personality disorder E. Obsessive-compulsive personality disorder

Dependent personality disorder

Patient presents as a 29 year old female who arrives at your office after a break-up with her boyfriend. She always did everything he wanted and cannot understand how he could not want to be with her. According to her, the boyfriend said she was "too clingy" and asked him to make every decision for her

Dependent personality disorder

A 24 year old woman with borderline personality disorder is admitted to a psychiatric hospital because of a suicidal ideation. The provider on call tells the patient about all the rules and regulations in the unit and that "although it is a great place to get better it is a lot of work." Which of the following is this provider attempting to do with this patient? A. Decrease idealization of the unit and the hospitalization B. Discourage the patient from splitting C. Dissuade the patient from signing in voluntarily D. Encourage the patient to seek admission elsewhere E. Investigate the patient's motivation for desiring admission

Decrease the idealization of the unit and the hospitalization

A 34-year-old woman presents to the clinic for evaluation after her family has raised concern over her behavior for the past three months. She states that she discovered the cure for cancer and will soon be famous. Her family reports that she has never had a scientific interest before and she now spends her time calling reporters instead of attending her job. She denies changes in mood or visual or auditory hallucinations. Which of the following is the most likely diagnosis? A Bipolar disorder B Brief psychotic disorder C Delusional disorder D Schizophrenia

Delusional disorder

The patient in Question 55.3 (narcissistic, therapist) has now been seeing a therapist twice weekly for the last year. The therapist and the patient have a good working alliance. During one therapy session, the therapist comes to the session 4 minutes late. He apologizes to the patient, stating that he had an emergency involving another patient. During the session, the patient notes that the therapist "isn't as sharp as some of the therapists I hear on the talk shows." Which of the following defense mechanisms is the patient using? A. Denial B. Devaluation C. Isolation of affect D. Rationalization E. Splitting

Devaluation

A 24 year old woman is seen in the emergency department after superficially cutting both her wrists. Her explanation is that she was upset because her boyfriend of 3 weeks just broke up with her. When asked about other relationships, she says that she has had numerous sexual partners, both male and female, but none of them lasted more than several weeks. Which type of psychotherapy might she be most likely to respond to? A. Dialectical behavioral therapy B. Interpersonal psychotherapy C. Parent assertiveness training D. Psychopharmacotherapy E. Supportive psychotherapy

Dialectical behavioral therapy

A 35 year old women is engaged in psychotherapy to address her avoidant personality disorder. In particular, she is distressed by her inability to maintain a romantic relationship with a man. During the course of treatment, the therapist learns that her father was an alcoholic and was physically abusive to the patient and her mother. Which defense mechanism best describes the patient's behavior? A. Undoing B. Splitting C. Isolation of affect D. Idealization E. Displacement

Displacement

An 11-year-old boy is admitted to an inpatient psychiatric unit for increasing aggression toward his peers and teachers at school. During evaluation, the patient states, "I am moody all the time." He has been aggressive toward his younger siblings and recently hit his dad during an argument. He has been diagnosed with ADHD and takes medications irregularly. At home, his mother reports that any small thing makes him erupt in extreme anger every other day for the last 2 years. Which of the following is the most likely diagnosis? A Oppositional defiant disorder B Disruptive mood dysregulation disorder C Intermittent explosive disorder D Major depression E Bipolar disorder—mania

Disruptive mood dysregulation disorder

A 10-year-old boy is being seen in the office for flight of ideas and inflated mood. He is diagnosed with mania without psychosis. Which of the following medications would be the best choice for mood stabilization for this patient? A Isotretinoin (Accutane) B Beclomethasone C Lithium D Divalproex E Risperidone

Divalproex

Which of the following neurotransmitters is affected most by first-generation antipsychotic medication used to treat patients with schizophrenia? A Acetylcholine B Dopamine C Glutamate D Norepinephrine

Dopamine

A 24-year-old woman presents to the clinic with excessive and persistent worry most days for the last six months. She states that the worrying disrupts her sleep, causes irritability, and makes her feel fatigued. Which of the following is the recommended first-line pharmacologic treatment? A Buspirone B Clonazepam C Duloxetine D Mirtazapine

Duloxetine (SNRI)

Your patient is a 25 year old man diagnosed with schizophrenia and has been doing well on clozapine. At his next office visit, he reports that his symptoms are returning. He has not missed any doses of clozapine and has not changed any other behaviors. You obtain a clozapine level and find that his level is therapeutic. What is the next line of treatment for this patient? A Add a benzodiazepine B Augment clozapine with haloperidol C Electroconvulsive therapy (ECT) D Switch from clozapine to another antipsychotic E Increase the dose of clozapine

Electroconvulsive therapy (ECT)

A 27-year-old woman with cyclothymia presents to the obstetrician for a routine prenatal visit. She has continued taking lithium during pregnancy. The risk of which congenital abnormality is increased in her child? A Fetal abdominal wall defect B Ebstein anomaly C Fetal renal dysplasia D Fetal spina bifida E Fetal tetralogy of Fallot

Ebstein anomaly

An elderly woman presents to the emergency department due to a hip fracture. She reports that she "hasn't been feeling very well" recently, and she is vague and hard to pin down regarding details. You think that there might be the odor of alcohol on her breath and suspect alcohol use disorder. Which of the following findings would be most supportive of your concern? A) A healed scar from a previous fall several years ago B) Microcytic anemia C) Elevated gamma-glutamyl transpeptidase D) Slightly elevated aspartate aminotransferase (AST), with normal alanine aminotransferase (ALT) E) Mini-Mental State Examination (MMSE) score of 28

Elevated gamma-glutamyl transpeptidase

Which of the following personality traits is most likely seen in patients with histrionic personality disorder? A. Callousness B. Emotional liability C. Recklessness D. Cognitive dysregulation E. Grandiosity

Emotional liability

Upon the urging of his supervisor, a 29 year old man decides to see a counselor at his employee assistance program. The patient was recently promoted to a position that requires increased interpersonal interaction as well as new supervisory responsibilities. Subsequently, his job performance has dropped off significantly. The patient states that since his transfer, he has been so nervous at work that he has not been able to think straight. He reports that his mood at home has been good, but he knows that he will fail at the new job because, "I have always been such a dope when it comes to working with other people." After several sessions, the counselor diagnoses the patient with avoidant personality disorder. Which of the following would be the most helpful in assisting the patient to manage his anxiety regarding his new job? A. Tell the patient that he needs to be more confident in his skills during this transition and "suck it up." B. Engage the patient in cognitive therapy to help him deal with his distorted thinking C. Give the patient a beta-blocker to help him control his anxiety D. Prescribe a benzodiazepine E. Tell the patient that he is probably not ready for this job if he is this anxious

Engage the patient in cognitive therapy to help him deal with his distorted thinking

A 25 year old with schizotypal personality disorder comes to his psychiatrist with a chief complaint of a depressed mood. He notes that since losing his job as an astrologer, he has been depressed and unable to sleep. He states that although his mood is usually fairly low (4 out of a possible 10), it has lately been a constant 2. The patient also notes problems with concentration and energy level and has experienced several crying spells. He reports he had premonitions and that certain foods could heal him, so he has been mixing "magical potions" and eating "magical foods." A mental status exam reveals an oddly dressed man with constricted affect, ideas of reference, unusual beliefs, and some mild paranoid. Which of the following medications is most likely to be helpful to this patient? A. Zolpidem for insomnia B. Divalproex sodium for mood disturbance C. Escitalopram for depressive symptoms D. Risperidone for paranoia E. Ziprasidone for ideas of reference

Escitalopram for depressive symptoms

A 23-year-old woman presents to the clinic complaining of depressed mood, irritability, increased appetite, breast tenderness, fatigue, and bloating that occurs monthly prior to the onset of menses. She states that the symptoms resolve after her menstrual period begins. The symptoms affect her productivity at work. What is first-line treatment for this condition? A Acupuncture B Combined oral contraceptive C Duloxetine D Fluoxetine

Fluoxetine

A 35-year-old female with c/o palpitations and chest pain for 2 days. She reports that she had been sweating with nausea and vomiting. She reports that she "checked her pulse which was about 156." She denies any personal or family history of heart disease or anxiety disorders. She rated her chest pain as 10/10 "which radiates down my left arm and up my jaw." ECG reads normal sinus rhythm.Troponin, CKMB, and other labs were normal. When she was told that her labs were normal, she flopped onto the bed and started "seizing." She stated, "I am seizing. Why won't you help me?" When the ER PA calls her out, she got angry and left against medical advice.

Factitious disorder

A 20-year-old woman presents to the clinic with excessive worrying for the past 7 months. She also reports insomnia, daytime fatigue, feeling on edge, and difficulty concentrating. Basic laboratory testing is performed and normal. Toxicology screen is negative. Which of the following is first-line pharmacologic treatment? A Alprazolam B Buspirone C Fluoxetine D Hydroxyzine

Fluoxetine

A 20-year-old woman presents with an excessive need to wash her hands for fear that she may have been in contact with contaminated objects. She spends more than one hour per day washing her hands, to the extent that this behavior is causing some social impairment. Which of the following therapies is the most appropriate for this patient? A Clomipramine B Fluoxetine C Lithium D Venlafaxine

Fluoxetine

A 23-year-old graduate student presents to her primary care provider complaining of severe abdominal cramps, bloating, and difficulty concentrating. She reports concern about her studies suffering over the last few days due to these symptoms. The patient recalls that these same troublesome symptoms occurred last month around the same time but resolved on their own after her period started. Her boyfriend accompanies the patient. When she steps out to use the restroom, the boyfriend confides in the clinician, "She has been so mean over the last few days. It's like anything I do or say sets her off." He does not recall any other changes in her behavior. Which of the following is the first-line treatment for these symptoms? A Fluoxetine B Lithium C Maprotiline D Methylphenidate E Spironolactone

Fluoxetine

A 25-year-old woman presents to the clinic with excessive worrying that she states has been occurring for years. She states that her excessive worrying is often about minor problems and causes insomnia, daytime fatigue, and difficulty concentrating. Her thyroid-stimulating hormone level is 1.0 mIU/L. She denies taking any medications or using any illicit substances. Which of the following is the most likely diagnosis? A Generalized anxiety disorder B Hyperthyroidism C Panic disorder D Phobia

Generalized anxiety disorder

A patient with schizoid personality disorder comes to his primary care provider with chief complaints of polyuria and polydipsia. He is found to have insulin-dependent diabetes. Which of the following interventions by the provider is likely to be most well received by this patient? A. Asking the patient to bring in a relative or close friend that he can describe the treatment regimen to both of them at the same time B. Referring the patient to a therapist for support in dealing with a chronic illness C. Giving the patient detailed written information about the disease and telling him that the provider will be available to answer any questions D. Referring the patient to a group that helps its members learn about diabetes and to better deal with their illness E. Scheduling frequent appointments with the patient is that all the treatment details can be explained on a one-to-one basis

Giving the patient detailed written information about the disease and telling him that the provider will be available to answer any questions

A 26-year-old graduate student is brought to the emergency department after becoming physically aggressive with his best friend. The patient has not been sleeping or eating well, resulting in a weight loss of approximately 10 lb, and has been "studying like a fiend" for his exams. The patient has no medical or psychiatric history. The patient's physical examination shows hypertension and tachycardia, as well as dilated pupils, diaphoresis, and a fine bilateral tremor in his hands. In the emergency department, he is oriented but belligerent and uncooperative. He states that his mood is fine, although he appears angry and tries to elope from the hospital. Several staff members are needed to control the patient, who is psychotic and extremely agitated, requiring placement in four-point restraints. Which of the following pharmacologic interventions is the most appropriate? A) Citalopram B) Diazepam C) Ascorbic acid D) Haloperidol E) Bupropion

Haloperidol

The patient in Question 45.1 becomes agitated and screams at the staff, claiming that they are trying to kill him and sell his organs to an organ trafficking syndicate. He appears internally stimulated and responding to imaginary voices. He begins throwing things at the treating team and threatening to beat anyone who enters his room, requiring him to be put into four-point restraints. Which of the following pharmacologic interventions is the most appropriate? A) Quetiapine B) Haloperidol C) Valproic acid D) Diazepam E) Bupropion

Haloperidol

A 39-year-old man with antisocial personality disorder, incarcerated for life after murdering a man, has a multitude of somatic complaints over the course of several years. Yearly physical exams never show anything physically wrong with him, yet he complains of a variety of aches and pains, neurologic symptoms, and GI distress. He does not enjoy the time he spends in the jail's infirmary. Speaking to the guards at the facility, there does not seem to be secondary gain from visiting the infirmary. Which of the following is the most likely explanation for the patient's complaints? A. He is malingering B. He has developed a psychotic disorder C. He has developed a somatic disorder D. He has an undiagnosed physical illness E. He has an undiagnosed anxiety disorder

He has developed a somatic disorder

A 16-year-old adolescent girl is incarcerated in a juvenile detention facility. She is currently charged with theft, apparently to support her and her boyfriend's drug habit. She has had multiple involvements with child and family services for running away from home, where she apparently had been sexually abused by her mother's boyfriend. She has a diagnosis of PTSD. Prior to the onset of the abuse, she was doing extremely well in school and was in an accelerated program. Which of the following factors speaks most strongly against making a diagnosis of antisocial personality disorder? A. Her concurrent diagnosis of PTSD B. Her gender C. Her age D. Antisocial acts committed to support a drug habit only E. Apparent high intelligence

Her age

Patient presents as a 27 year old female who arrives at your office in a short skirt and a very revealing top. At the beginning of the interview, she flirts with you and asks you if you like her dress. You overheard her yelling into her cell phone in the waiting room which later turned to tears. She describes her flu symptoms as if she's "dying"

Histrionic personality disorder

A 25-year-old woman presents to the clinic with concerns about mood swings. She states she gets into bad fights with her boyfriend every month and has almost ended their relationship a few times. Around this time, she is more tearful, anxious, and irritable. Which history question would help to confirm the suspected diagnosis? A Are you afraid of being alone or abandoned? B Do you experience more impulsivity during these times? C Do you feel the need to escape during these times? D How does the timing of these fights correlate with your menstrual cycle?

How does the timing of these fights correlate with your menstrual cycle?

A 25-year-old male graduate engineering student who is at your office for the fourth time in 1 week to be sure he does not have chlamydia. Despite having one sex partner, he learned there is an increase in the incidence of STDs on campus. He reports that he has only had sex once but learned that chlamydia is hard to culture. He reports he may have dysuria intermittently but is not sure. He has been evaluated at each visit, and physical and laboratory examination has been completely normal each time.

Illness anxiety disorder (hypochondriasis) / treatment = group/insight-oriented therapy, medications (SSRIs) if concurrent/underlying anxiety or major depressive disorder

A 32-year-old man with a long-standing heroin addiction has recently started maintenance treatment with methadone. Three days since starting the methadone regimen, he is now experiencing some craving, diarrhea, and mild sweating. His urine toxicology screen is negative for any opiates besides methadone. Which of the following is the most appropriate course of action? a. Increase the dose of methadone b. Decrease the dose of methadone c. Keep the dose of methadone the same and assure the patient that d.the symptoms will subside d.Begin clonidine to be taken in addition to the methadone.e.Put the patient on a 1-week methadone taper program and refer him to Narcotics Anonymous.

Increase the dose of methadone

A 16 year old male with a long history of arrests for breaking and entering, assault and battery, and drug possession is found dead in his room. For several months he had been experiencing headaches, tremors, muscle weakness, unsteady gait, and tingling sensation in his hands and feet. An empty CVS plastic bag was found beside his body. These symptoms and the manner of his death suggest that he was addicted to which of the following substances.

Inhalant

A 24 year old woman is called into the head office of the agency where she works and told that her chronic lateness in completing her assignments will result in her dismissal if she does not change her behavior. The patient really loves her job, and the news comes as a major blow. That night at home, she tells her boyfriends in great detail about each and every step of the meeting and spends the entire night thinking about her job. They boyfriends tells her that she does not "look" particularly upset. Which of the following defense mechanisms is being used by this woman? A. Undoing B. Displacement C. Intellectualization D. Rationalization E. Splitting

Intellectualization

A woman with schizoid personality disorder was involved in a motor vehicle accident in which she was rear-ended by another car. The driver of the other car refused to take responsibility for the accident and hired a lawyer to provide his defense. The woman spends hours every day thinking about the specifics of the accident, including such details as the color of the cars involved and what each party to the accident was wearing. Which of the following defense mechanisms, common to patients with schizoid personality disorder, is the woman using? A. Sublimation B. Undoing C. Projection D. Intellectualization E. Introjection

Intellectualization

A 25-year-old man presents to the psychiatrist for evaluation. He has a history of a hypomanic episode and several major depressive episodes and is currently experiencing a depressive episode. Which of the following is an appropriate pharmacologic treatment for this individual's depressive episode? A Duloxetine B Fluoxetine C Lamotrigine D Venlafaxine

Lamotrigine

A 24-year-old man has been diagnosed with bipolar disorder type I and is being started on a new medication that is known to cause tremors, hypothyroidism, weight gain, and nephrogenic diabetes insipidus. Which of the following medications is being started? A Carbamazepine B Lithium C Olanzapine D Valproic acid

Lithium

A 54-year-old man is being seen for anxiety associated with depression. He has been tried on various medications without success, and the psychiatrist is considering prescribing a benzodiazepine. He also has been diagnosed with chronic hepatitis C infection. Which agent would be safest for this patient? A) Chlordiazepoxide B) Clonazepam C) Alprazolam D) Diazepam E) Lorazepam

Lorazepam

A 58-year-old man with hypertension and hyperlipidemia is admitted to the general medicine unit for intravenous antibiotics for treatment of pneumonia. After 2 days he becomes increasingly anxious, with complaints of "shaking" and sweating. His vitals demonstrate a temperature of 100.4 °F, blood pressure of 170/97 mm Hg, pulse of 110 beats per minute, and respiratory rate of 16 breaths per minute. On examination he appears diaphoretic and flushed, with a coarse tremor of his upper extremities bilaterally. Laboratory test results are significant for slightly elevated aspartate aminotransferase (AST), alanine aminotransferase (ALT), and gamma-glutamyl transferase (GGT). Review of his chart reveals a long history of daily heavy alcohol use. When confronted with this information, the patient reluctantly admits to drinking "one fifth" of vodka plus several beers daily, with little sobriety. His last drink was the day prior to admission. Administration of which of the following medications would be the most appropriate for this patient? A) Acamprosate B) Disulfiram C) Lorazepam D) Naltrexone E) Thiamine

Lorazepam

After a thorough evaluation, your new patient is diagnosed with specific phobia related to flying. She has agreed to start cognitive-behavioral therapy, but because flying is required as part of her work, you also decide to prescribe a short-term rescue medication. Which of the following medications would you prescribe as a first-line treatment? a. Escitalopram b. Lorazepam c. Modafinil d. Quetiapine

Lorazepam

A 44-year-old woman comes to your office for a follow-up visit. She recently received a diagnosis of major depressive disorder and began treatment with citalopram (an SSRI) 6 weeks ago. She claims to feel "happy again," without further depression, crying spells, or insomnia. Her appetite has improved, and she has been able to focus at work and enjoys time with her family. Although she experienced occasional headaches and loose stools at the beginning of her treatment, she no longer complains of any side effects. Which of the following is the most appropriate next step in her treatment? A Consider a different class of antidepressants. B Discontinue the citalopram. C Increase the dose of citalopram. D Lower the dose of citalopram. E Maintain the current dose of citalopram

Maintain the current dose of citalopram

A 20 year old woman comes to see a psychiatrist at the insistence of her mother, who states that her daughter just "isn't herself." The patient is spending a great deal more time alone in her bedroom, she doesn't seem to be caring for her hygiene as well as usual, and has been missing work a great deal more. She is very clingy and attention seeking with her mother. Which of the following diagnoses best fits the patient's presentation? A. Histrionic personality disorder B. Borderline personality disorder C. Bipolar disorder, mania D. Major depressive disorder E. Delusional disorder

Major depressive disorder

A 46-year-old man presents to his primary care clinic with concerns about insomnia for two months. While interviewing the patient, he also mentions that he has not been eating as much lately, feels tired all the time, has trouble focusing at work, and does not feel like meeting up with his friends or family anymore. He reports these symptoms started around the same time as his insomnia. His physical exam is unremarkable, and vital signs are all within normal limits. Which of the following is the most likely diagnosis? A Bipolar II disorder B Cyclothymia C Dysthymia D Major depressive disorder

Major depressive disorder

A 79-year-old Army veteran with an extensive medical history presents to his primary care provider for cerumen irrigation and states "I am no value to anyone doc...why didn't they kill me in Korea?" For the past year, the patient reports an "unrelenting" loss of interest in living, reoccurring guilt regarding his military service, hopelessness, terminal insomnia, decreased appetite, decreased energy, decreased motivation, and low spirits. He denies contemplating suicide but states "but I am ready to go!" A comprehensive suicide assessment reveals that the patient is at moderate risk for suicide. Which of the following is the most likely diagnosis? A Adjustment disorder B Generalized anxiety disorder C Major depressive disorder D Persistent depressive disorder

Major depressive disorder

A 40-year-old gravida 3, para 2 banker at 33 weeks' gestation is referred to psychiatry by her obstetrician after the patient did not make appropriate weight gains during the pregnancy. The patient admits that she has had depressive symptoms, irritability, and suicidal thoughts "on and off" since the birth of her first child. She reports that the symptoms worsened again after the delivery of her second child. Her husband recently left her, claiming she was "always PMSing." For the last few months, she has struggled to raise the children as a single parent. She denies any plan to hurt herself, her other children, or the fetus. She voices her concern about caring for her children and the newborn after delivery. What is the most likely diagnosis? A Borderline personality disorder B Bipolar disorder C Major depressive disorder, with peripartum onset D Posttraumatic stress disorder E Premenstrual dysphoric disorder

Major depressive disorder, with peripartum onset

The patient in Question 14.1 is evaluated fully and is determined to have "double depression." What is meant by the term "double depression"? A Two episodes of major depressive disorder per month consistently B Superimposed bipolar II disorder and persistent depressive disorder C Major depressive episode superimposed on persistent depressive disorder D Two family members suffering from major depressive disorder concurrently E Persistent depressive disorder with current symptoms twice as disabling as usual

Major depressive episode superimposed on persistent depressive disorder

Twelve hours after a surgical admission for a broken arm, a 42-year-old woman begins to complain of feeling jittery and shaky. Six hours later, she tells staff members that she is hearing the voice of a dead relative shouting at her, although on admission she denied ever having heard voices previously. She complains of an upset stomach, irritability, and sweating. Her vital signs are BP 150/95 mm Hg, pulse 120 beats per minute (bpm), respirations 20 breaths per minute, and temperature 100 °F (37.8 °C). The patient reports no prior significant medical problems and says that she takes no medications. She has not had prior complications due to general anesthesia. Most likely diagnosis: ? Next step in treatment: ? What should be administered prior to glucose: ?

Most likely diagnosis: Alcohol withdrawal Next step in treatment: benzodiazepine What should be administered prior to glucose: Thiamine

The patient in the previous two questions is treated in the inpatient psychiatric unit with antipsychotic medications, and his symptoms are well controlled after a week of inpatient stay. The patient is still ambivalent about his substance use problem. Which of the following would be the next recommended step in management? A) Motivational interviewing B) Naltrexone C) Refer to the college counselor for follow-up D) Allow the patient his right to continue smoking marijuana E) Drug counseling

Motivational interviewing

Which of the following statements is accurate regarding narcissistic personality disorder? A. NPD typically does not manifest until mid-adulthood (age > 30 years) B. The incidence of NPD is equal between men and women C. NPD is frequently associated with comorbid axis I diagnoses D. In general, NPD lessens over time as the individual ages

NPD is frequently associated with comorbid axis I diagnoses

The patient in Question 38.1 (alcohol abuse) is treated appropriately, with normalization of his vitals and laboratory test results, and he is released from the hospital after 8 days. He immediately enrolls in an outpatient rehabilitation program, which he is required to attend three times per week, as well as AA meetings daily. He is quite hopeful about his recovery and denies significant depression, although he is greatly concerned about a possible relapse. While he is interested in medication to minimize his risk, he admits to being "very forgetful" with medications. Which of the following medications would be most appropriate to initiate in this patient? A) Acamprosate B) Disulfiram C) Lorazepam D) Naltrexone E) Sertraline

Naltrexone

Patient presents as a 45 year old male lawyer being worked up for low back pain. He demands that he sees the chief neurosurgeon and won't let any staff in the room. He says he knows "very important people" and that there will be "trouble" if he waits longer than 5 minutes in the waiting room

Narcissistic personality disorder

A 50-year-old man with a past history of chronic, treatment-resistant schizophrenia was admitted last night after reemergence of command AHs telling him to "do bad things." He had been recently hospitalized and stabilized on clozapine. He denies missing any doses. What addiction is the most common form of substance abuse in patients with schizophrenia and likely contributed to the patient's recent psychotic episode? A Alcohol B LSD C Marijuana D Nicotine E Phencyclidine (PCP)

Nicotine

A 46 year old man presents to a primary care physician with a chronic cough. He reports that he has not seen a physician in 25 years, but he has not been sick during this time. He lives in a cabin in the woods, and although he does not have any friends, he reports that he does not feel lonely. He is well-groomed, dressed normally, and does not have any bizarre beliefs. He describes his mood as "fine," and his speech is fluid, albeit limited in amount. His thought process is linear, and his thought content is appropriate. He denies any visual or auditory hallucinations. Which of the following is true about this patient's psychiatric condition? A. It has a high correlation with schizophrenia B. The patient should be treated with an antidepressant C. No specific treatment is needed at this time D. The patient is likely very responsive to criticism and praise E. The patient likely suffers from a comorbid substance use disorder

No specific treatment is needed at this time

A 25-year-old woman who is a business analyst from Colorado comes to her primary care provider's office for a wellness check. She reports using marijuana occasionally ever since her college days at parties and on the weekends, maybe once or twice a month. She likes the feeling it produces but denies any problems arising from it. She denies other substance use except an occasional glass of wine on the weekend. She is highly regarded at work and recently got engaged to her high school sweetheart. She wants to settle down and start a family and feels her "partying days" are over. What is the likely diagnosis in this patient? A) Cannabis-induced depression B) Cannabis use disorder C) Cannabis withdrawal syndrome D) Non-problematic cannabis use E) Polysubstance use disorder

Non-problematic cannabis use

A 26 year old woman comes to see a psychiatrist because she has been taking showers for 6-7 hours every day. She explains, "It all starts when I wake up. I am sure I am covered in germs, and if I don't wash, I will get sick. If I don't wash, I am paralyzed with anxiety. Once I'm in the shower, I have to shower in a particular order. If I mess up, I have to start over, and this takes hours and hours. My skin is cracking and bleeding because I spend so much time in the water." Which of the following conditions does this patient most likely have? A. OCD B. Obsessive-compulsive personality disorder C. Obsessive-compulsive traits D. Paranoid personality disorder E. Schizoid personality disorder

OCD

A 32-year-old man presents to the clinic accompanied by his girlfriend. She describes episodes wherein the patient checks every door in their house exactly three times before they leave to ensure they are all closed. She states this happens every time they leave their house. When questioned about this behavior, the patient replies he "feels a need" to check every door and cannot leave the house without fulfilling the need. Which of the following disorders is the most likely diagnosis? a. Generalized anxiety disorder b. Obsessive-compulsive disorder c. Obsessive-compulsive personality disorder d. Tic disorder

Obsessive-compulsive disorder

Patient presents as a 26 year old PA student at the student health center with the complaint of being unable to sleep. Although he is a very successful student, over the past few months he has become increasingly preoccupied with failing. The patient states that he wakes up 10-15 times per night to check his textbooks for factual recall. He has tried unsuccessfully to suppress these thoughts and actions, and he has become extremely anxious and sleep-deprived. He has no past medical history and family history is significant for a parent with Tourette's syndrome. He is started on a first-line medication for his disorder, but after eight weeks of use, it is still ineffective.

Obsessive-compulsive personality disorder

A 23 year old medical student makes lists of all the tasks he must accomplish each day. He spends hours studying and refuses to go out with his colleagues even when there are no tests on the immediate horizon, preferring to spend his time looking at specimens in the laboratory. He keeps meticulous notes during all his classes and attends every lecture, not trusting his colleagues to take notes for him. He is doing well in school and has a girlfriend who is also a medical student. Which of the following conditions does this student most likely have? A. OCD B. Obsessive-compulsive personality disorder C. Obsessive-compulsive traits D. Schizoid personality disorder E. Paranoid personality disorder

Obsessive-compulsive traits

A 28-year-old man is brought to a psychiatrist complaining that he has been hearing voices for the past several weeks. He says that he also heard these voices 3 years ago. He notes that his mood is "depressed" and rates it 3 on a scale of 1 to 10 (with 10 being the best he has ever felt). He does not recall if his mood was depressed the last time he had psychotic symptoms. Which of the following actions should the provider take next? A Obtain more detailed information about the time course of the psychotic symptoms and the mood symptoms. B Treat the patient with an antipsychotic agent. C Treat the patient with an antidepressant medication. D Request a urine toxicology screening. E Refer the patient to supportive psychotherapy

Obtain more detailed information about the time course of the psychotic symptoms and the mood symptoms

A 20-year-old man presents to the psychiatric emergency center with hallucinations and delusions. On exam, he demonstrates blunted affect, apathy, and circumstantiality. Chart review shows that he has had symptoms for at least six months. Toxicology screen is negative. He is diagnosed with schizophrenia. Which of the following is the best treatment for the management of this acute psychotic episode? A Clozapine B Fluoxetine C Lorazepam D Olanzapine

Olanzapine

A 30-year-old man has a history of hypomanic episodes and major depressive episodes. He is diagnosed with bipolar disorder type 2. Which of the following is the most efficacious treatment for an acute hypomanic episode? A Fluoxetine B Lithium C Olanzapine D Valproate

Olanzapine

Parents of a 10-year-old boy note that their son does well with his family until he is not allowed to do something he wants to do. When this occurs, he will get irritable, impulsively aggressive, and agitated for several hours. Once he calms down or gets his way, he is happy and pleasant again. At school, he has no trouble focusing, but if he does not want to do something, he becomes argumentative. Which of the following is the most likely diagnosis? A Oppositional defiant disorder B Bipolar disorder—mania C Attention deficit hyperactivity disorder combined type D Major depression E Disruptive mood dysregulation disorder

Oppositional defiant disorder

A 16-year-old adolescent boy is being seen for follow-up after a diagnosis of schizoaffective disorder based on the findings of depressive symptoms and hearing voices. The patient is prescribed olanzapine. Upon returning to the office after 1 week, he has not improved. In fact, he seems to have worsened. Which of the following is your best next step? A Increase the olanzapine. B Add another neuroleptic. C Add a mood stabilizer. D Add an antidepressant. E Order a urine or serum toxicology for substances of abuse

Order a urine or serum toxicology for substances of abuse

A 23-year-old woman presents to the emergency department with chest pain and palpitations that have resolved since her presentation. She was stuck in traffic when she noticed her heart racing accompanied by sweating, chest pain, and dizziness. She was extremely fearful because she has never felt like this before. There is no significant family or personal medical history. Physical examination, electrocardiogram, and laboratory studies are normal at this time. Which of the following is the most likely diagnosis? a. Agoraphobia b. Myocardial infarction c. Panic attack d. Panic disorder

Panic attack

A 22-year-old woman presents to the clinic concerned about recent episodes of intense fear and discomfort. She has had several of these attacks that reach a peak intensity within minutes and last less than an hour. During these episodes, she experiences chest pain and palpitations. Although she reports some concern about having future panic attacks, the patient states she generally does not feel anxious. Which of the following is the most likely diagnosis? A Generalized anxiety disorder B Illness anxiety disorder C Panic disorder D Somatic symptom disorder

Panic disorder

A 36 year old man comes to his primary care provider's office with the chief complaint that "people are out to hurt me." Despite being reassured by his wife that this is untrue, the patient is convinces that people are observing his behavior and actions at home and at work, using telescopic lenses and recording devices. He has torn apart his office on more than one occasion, "looking for bugs." The patient's wife says that this behavior is relatively new, appearing somewhat suddenly after the patient was robbed on the way to his car approximately 6 months previously. Which of the following symptoms best describes what the patient is experiencing? A. Ideas of reference B. Hallucinations C. Paranoid delusions D. Paranoid ideations E. Thought disorder

Paranoid delusions

Patient presents as a 33 year old female who says that her coworkers are planning to have her fired and they often ridicule her when she is not there. She is attempting to pursue legal action to get her coworkers reprimanded for this behavior as she knows that they have an intension to harm her in some way

Paranoid personality disorder

Which statement best characterizes the difference between patients with avoidant personality disorder and those with schizoid personality disorder? A. Patients with avoidant personality disorder have fewer friends than those with schizoid personality disorder B. Patients with avoidant personality disorder have higher self-esteem than those with schizoid personality disorder C. Patients with avoidant personality disorder would like to have friends more than patients with schizoid personality disorder D. Patients with avoidant personality disorder are better at accepting criticism than patients with schizoid personality disorder E. Patients with avoidant personality disorder are less anxious than are patients with schizoid personality disorder

Patients with avoidant personality disorder would like to have friends more than patients with schizoid personality disorder

What is the main difference between obsessive-compulsive personality disorder and OCD? A. Obsessive-compulsive personality disorder afflicts more women than men, whereas OCD afflicts more men than women B. People with obsessive-compulsive personality disorder do not experience as much social or occupational impairment as those with OCD C. People with obsessive-compulsive personality disorder don't have true obsessions or compulsions like people with OCD D. People with OCD are not as conscientious about their rituals as people with obsessive-compulsive personality disorder

People with obsessive-compulsive personality disorder don't have true obsessions or compulsions like people with OCD

A 65-year-old retired engineer presents to his primary care provider at the insistence of his wife. The patient reports not needing as much sleep as he used to—only a few hours for the last few nights. He is considering going back to work because he wants to share his knowledge with "all the young fledgling engineers—they need my guidance." His wife notes that he has been very irritable and overly talkative lately. The patient has no previous psychiatric or medical history. He denies use of drugs or alcohol. Which of the following is the best next step? A Admit to the inpatient psychiatric unit B Order a urinalysis C Perform a physical examination D Start a mood-stabilizing drug E Start an antipsychotic drug

Perform a physical examination

A 32-year-old woman presents to the clinic with depressed mood for the past two and a half years. She states that she has been sleeping excessively, had poor appetite, and experienced a lack of concentration during this time. Which of the following is the most likely diagnosis? A Bipolar disorder type 2 B Cyclothymic disorder C Persistent depressive disorder D Unipolar depression

Persistent depressive disorder

A 25-year-old man presents with depressive symptoms after witnessing the death of his brother in a motor vehicle collision four months ago. The patient reports two months of repetitive distressing dreams about the event and frequent dissociative reactions when he is a passenger in a vehicle. Which of the following is the most likely diagnosis? A Acute stress disorder B Generalized anxiety disorder C Obsessive-compulsive disorder D Post-traumatic stress disorder

Post-traumatic stress disorder

Which of the following medications can treat nightmares associated with post-traumatic stress disorder? a. Paroxetine b. Prazosin c. Trazodone d. Venlafaxine

Prazosin

A 25-year-old woman presents to the clinic complaining of cyclical irritability, anxiousness, breast tenderness, fatigue, and bloating. She says the symptoms begin five days before her menstrual cycle and resolve within two days of her starting menstruation. In addition, she explains that these symptoms have a significant impact on her work and social life. Which of the following is the most accurate diagnosis? A Generalized anxiety disorder B Premenstrual dysphoric disorder C Premenstrual syndrome D Unipolar depression

Premenstrual dysphoric disorder

Which of the following is the strongest risk factor for suicide? A Age group 30 - 50 years B Previous suicide attempt C Psychiatric disorders D Sexual minority

Previous suicide attempt

A 42 year old woman undergoing psychotherapy storms into her therapist's office for her session and angrily accuses the therapist of "trying to undermine [her] intelligence." After a discussion with the therapist, it becomes clear that it is the patient who is second-guessing herself, thereby "undermining" her own intelligence. Which of the following defense mechanisms is this patient using? A. Denial B. Identification with the aggressor C. Intellectualization D. Projection E. Reaction formation

Projection

A 27 year old man is referred to your office by his boss for filing numerous unfounded complaints against his co-workers. The patient tells you "there is nothing wrong with me! In fact, everyone at work is trying to sabotage me constantly! They are the ones who should be sitting here!" He denies any hallucinations or delusions. Which of the following is the best treatment option? A. Prescribe an antipsychotic B. Prescribe an antidepressant C. Provide supportive psychotherapy D. Provide provocative psychotherapy E. Prescribe an anxiolytic

Provide supportive psychotherapy

A 44 year old women presents to your clinic for anxiety during social functions at her office. As a manager, she is expected to attend these events. You learn that her anxiety occurs only in social situations outside her small circle of friends. She tells you, "People keep looking at me because I'm not worthy of my role." She does not have discrete panic attacks, and her anxiety does not occur at other times. What is the best treatment for this patient? A. Prescribe an SSRI B. Prescribe a benzodiazepine to take before attending social events C. Psychodynamic therapy D. Exposure therapy E. Prescribe a beta-blocker to take before attending social events

Psychodynamic therapy

A 30-year-old woman presents to the clinic stating she has been in a depressed mood for the past month. She states that other symptoms include inability to sleep, decreased interest in normally pleasurable activities, feeling guilty, lack of concentration, and increased appetite. She denies prior symptoms of hypomania or mania and denies suicidal ideation. Which of the following is the best treatment? A Mirtazapine B Psychotherapy C Psychotherapy and sertraline D Sertraline

Psychotherapy and sertraline

A 20-year-old male college student was reported by others for spying through the windows of his fellow female dormmates.

Voyeuristic disorder

A paraphilic disorder in which sexual arousal is derived from observing unsuspecting individuals undressing or naked

Voyeuristic disorder

A 22-year-old woman is referred to your office by her family care provider for evaluation of depression. Her primary care provider is unsure whether she is suffering from persistent depressive disorder or a major depressive disorder. Which of the following characteristics is more consistent with persistent depressive disorder than major depression? A Episodic course B Numerous neurovegetative symptoms C Presence of psychotic symptoms D Severe impairment in functioning E Symptoms are of a low intensity

Symptoms are of a low intensity

Of the following features, which one is most suggestive of the diagnosis of Schizophrenia? A disorganized speech B delusions C hallucinations D the presence of both positive and negative symptoms E disorganized or catatonic behavior

the presence of both positive and negative symptoms

A 32 year old man was found sleeping on a subway vent. Physical exam shows a BP of 90/50 with HR 135. He has bilateral nystagmus, but pupils are normal. He is ataxic and cannot stand for longer than a few seconds. During the exam he does not speak spontaneously and he sometimes falls asleep but is easily aroused. His answers to questions are minimal and some questions are not answered at all. He is disoriented and confused. You are unable to complete a mental status exam because he is unable to cooperate fully. What is the most likely diagnosis?

Wernickes syndrome

Are men or women more likely to suffer from dissociative identity disorder?

Women

A 55-year-old man presents to the clinic with his daughter who states that her father was in a traumatic car collision several years ago. Since that time, he refuses to ride in cars or drive due to the fear of getting in another collision. The patient's daughter explains that he experiences significant distress and panic during car rides and avoids discussing it. She is concerned that her father's well-being is significantly impacted, as he almost never leaves the house. Which of the following is the most likely diagnosis? a. Agoraphobia b. Panic disorder c. Post-traumatic stress disorder d. Specific phobia

Specific phobia

A 9-year-old boy has been refusing to leave home. He complains of abdominal pain, nightmares, and fear of getting hurt and losing his mother. Five weeks ago, his mother was in a car accident. He is diagnosed with separation anxiety disorder, and fluoxetine treatment is initiated. The Food and Drug Administration (FDA) recommends the clinician monitor closely for which of the following? A) Hypovolemia B) Suicidal thoughts C) Anorexia D) Hypertension E) Delusions

Suicidal thoughts

A 24-year-old woman with a diagnosis of panic disorder is being treated with alprazolam 1 mg twice daily in addition to citalopram 20 mg daily. Since starting the medications, she has noticed a reduction in the number of panic episodes but reports feeling anxious and close to a panic attack when she wakes up each morning, which lasts until she takes her morning dose of medications. What is the next best step in management? A) Increase the dose of alprazolam B) Increase the dose of citalopram C) Switch to an Serotonin-norepinephrine reuptake inhibitor (SNRI) like venlafaxine D) Switch from alprazolam to clonazepam E) Add buspirone

Switch from alprazolam to clonazepam

Which of the following is indicated in the treatment of major depressive disorder? A Cannabidiol B Ketogenic diet C Repetitive transcranial magnetic stimulation D Transorbital lobotomy

Repetitive transcranial magnetic stimulation

A 35 year old woman who has been diagnosed with a histrionic personality disorder has seen her psychotherapist once a week for the past year. She has come in the last few visits subtly different. She appears more distracted, is late for appointments, reports an increased amount of arguments with her family, and appears flushed and even sweaty. You asked about use of illicit substances and she denies it. Which of the following is the most important step in this patient's management? A. Breaking confidentiality and calling e family member to discuss this change B. Request a urine toxicology to screen for substance abuse C. No further workup is necessary; continue with the current framework of psychotherapy and treatment D.Ask her to go to her primary care provider E. Refer her to an Alcoholics Anonymous group

Request a urine toxicology to screen for substance abuse

A 40-year-old man with schizoaffective disorder has been hospitalized in an inpatient psychiatry unit for the third time in the last 5 years. During each episode prior to hospitalization, he becomes noncompliant in taking his medications, develops manic symptoms and auditory hallucinations, and then becomes violent. In the inpatient unit, he physically threatens other patients and staff and is generally agitated. He is put in isolation to help quiet him. The patient is prescribed a mood stabilizer. Which of the following medications would be most appropriate to help relieve this patient's acute agitation? A Buspirone B Fluoxetine C Chloral hydrate D Risperidone E Benztropine

Risperidone

A 26-year-old man presents with major depressive episodes, delusions, and hallucinations. He started hearing voices six months ago, and his depressed mood has been present for the past three weeks. Physical exam reveals no abnormality. Laboratory studies and urine drug screens demonstrate no abnormalities. Which of the following is the most likely diagnosis? A Delusional disorder B Schizoaffective disorder C Schizophrenia D Schizophreniform disorder

Schizoaffective disorder

A 36-year-old woman with a past psychiatric history of bipolar disorder (diagnosed at age 28) and schizophrenia (diagnosed at age 30) presents to the emergency department reporting "voices that laugh about my dad raping me...raping me... which makes me want to thrill myself." She is nonadherent to lurasidone and is admitted to inpatient psychiatry. During her initial psychiatric assessment, the patient notes depressed mood and decreased motivation to get out of bed for the past month. She also notes intense guilt, decreased appetite, sluggish movement, and inability to concentrate amid thoughts of suicide. On mental status exam, she appears disheveled and older than her stated age, has atonal speech (with frequent clangs), describes her mood as "depressed-a-mess," displays blunt affect, loose thought process, and momentarily responds to an absent "voice" during interview. What is the primary diagnosis for this patient? A. Major depressive episode with psychotic features B. Mixed episode of bipolar disorder C. Schizoaffective disorder D. Schizophrenia with comorbid major depression

Schizoaffective disorder

A 30 year old man is brought to a psychiatrist by his parents because they are concerned that he seems to lack ambition. He lives in his parents' basement, never having lived on his own. He has no friends or social contacts. He works from home as a computer programmer, working for the same company since graduating from college online. When asked, he shrugs and tells you he is happy and doesn't need anything else out of life. Which of the following is the most likely diagnosis? A. Avoidant personality disorder B. Dependent personality disorder C. Schizotypal personality disorder D. Schizoid personality disorder E. Autistic spectrum disorder

Schizoid personality disorder

Patient presents as a 50 year old man who lives alone in a cabin on several acres of land. He rarely interacts with his family or community other than when he comes to town once a month to pick up supplies.

Schizoid personality disorder

A 20-year-old female presents to the emergency room via ambulance. She was found by the police walking naked in the woods behind her home. Her speech displays minimal thoughts and is disorganized and she appears to be responding to internal stimuli. Further questioning reveals she feels that she is being watched by aliens. Her family provided further history that she has been acting bizarre for 2 months now. What is the most likely diagnosis? A Schizophrenia B Schizophreniform disorder C Schizoaffective disorder D Schizotypal personality disorder

Schizophreniform disorder

A 20 year old man is brought to a psychiatrist by his parents for odd thinking. He is dressed in clothes consistent with a 1960s hippie, with long unruly hair and marginal hygiene. He was recently fired from his job for not showing up for his shifts and was forced to move back in with his parents. He has artistic aspirations and is very interested in philosophy, metaphysics, magic, and the occult. He talks about his desire for fame and wealth, given his special talents. He has recently gotten into some legal trouble because he produced artwork of fanciful paper currency, which he attempted to use at some local stores. However, he admits that he did not think that this was going to work and describes this as performance art. Which of the following is the most likely diagnosis? A. Schizophrenia B. Schizoid personality disorder C. Schizotypal personality disorder D. Delusional disorder E. Bipolar disorder

Schizotypal personality disorder

Patient presents as a 47 year old female has recently quit her job in order to start a small business. She claims that the spirits have told her to sell trinkets each of which has a unique power. She is known for dressing oddly and due to her strange behavior has trouble making friends.

Schizotypal personality disorder

A 24-year-old man is diagnosed with post-traumatic stress disorder after experiencing a motor vehicle collision. He endorses flashbacks, insomnia, and an intense fear of getting in another vehicle. He avoids being in motor vehicles and, as a result, has missed work for six weeks since the incident. Which of the following medications is most appropriate in treating this patient? a. Aripiprazole b. Clomipramine c. Clozapine d. Sertraline

Sertraline

A 25-year-old woman presents to the clinic with recurrent episodes of abrupt and intense fear. She says the episodes last about 10 minutes each time. She states that she does not want to go into public places because she fears she will have one of these episodes. She has had extensive medical evaluation without an identifiable etiology and reports no substance use. Which of the following is considered first-line pharmacologic therapy? a. Gabapentin b. Lorazepam c. Mirtazapine d. Sertraline

Sertraline

A 23-year-old male who concerned that his behavior has resulted in his inability to maintain a relationship. He reveals that he requires his partners to strangle him and humiliate him in order for him to achieve and maintain an erection.

Sexual masochism disorder

Which of the following would be the most useful psychiatric treatment for the patient in Question 53.1? (dependent disorder) A. Antianxiety medication B. Antidepressant medication C. Nothing at this time because research suggests personality disorders improve over time D. Individual psychotherapy E. Sociotherapies

Sociotherapies

A 25-year-old female who presents today with multiple complaints that have been ongoing for more than 6 months. She reports that "it all started about 10 months ago with pain in my neck, shoulders, back, legs, and feet." She denies any trauma. There is no family history of juvenile rheumatoid arthritis or osteoarthritis. She stated that the pains do not respond to treatments, and they "just come and go making it difficult to hold a job." She is constantly worried about her symptoms. Now, she has a headache, abdominal pain, bloating, and "some seizures." She previously had seen a headache specialist, gastroenterologist, and obtained a number of electrocardiograms in the emergency department. Their respective thorough workup was negative. Her mother had similar episodes as well. On physical exam, you note an anxious woman with a depressed affect. No significant physical exam findings are noted. Labs/imaging are all within normal range.

Somatic symptom disorder

You are consulted to evaluate a 45 year old married woman who was admitted to the surgical service 2 days ago for an appendectomy. The procedure went well, but she was found to be tearful, stating, "I wish I were dead." On obtaining further history, she is quite cooperative and talkative. She is questioned about her earlier comments, and she states that she "wanted attention, I guess." She is upset that her husband is not with her in the hospital; she has "never been away from him" for this long since they started dating when the patient was 16 years old. She feels helpless and is having a difficult time being active in her care. She feels overwhelmed regarding her postsurgical and discharge instructions, and the nursing staff has become frustrated with her constant need for reassurance. Although at times she is tearful during the interview, she denies prior to recent pervasive depressive or neurovegetative symptoms and is not actively suicidal. Which of the following is the most appropriate approach to this patient? A. Encourage her to learn more about her surgery and become more proactive in her care B. Persuade her to become less dependent on her husband C. Insist her husband be present at all times while his wife is hospitalized D. Spend regular, short periods of time with her to discuss discharge planning and aftercare E. Transfer her to the psychiatric unit

Spend regular, short periods of time with her to discuss discharge planning and aftercare

The patient in question 57.1 (borderline personality disorder) complains to her psychiatrist that all the nurses on the floor don't know what they are doing and are rude. She later tells her nurse she is the best nurse on the floor and wished her psychiatrist cared about her the way her nurse did. Which of the following defense mechanisms is this patient using? A. Altruism B. Intellectualization C. Splitting D. Sublimation E. Undoing

Splitting

A 22 year old man with borderline personality disorder loses his job at a local restaurant, the first job he has held for longer than a month. His mother dies suddenly 3 weeks later. One month after his mother's death, the patient tells his therapist, whom he has been seeing once a week, that he has trouble sleeping, waking up at 3am and then unable to go back to sleep. He has lost 13 lb in 5 weeks without trying to do so. He reports low energy and a decreased interest in his usual hobbies. He states that he feels depressed but then grins and says, "But I'm always depressed, aren't I?" Based on his history, which of the following should the clinician do next? A. Ask the patient to keep a sleep log B. Begin seeing the patient for daily psychotherapy C. Hospitalize the patient D. Start treating the patient with a mood stabilizer such as carbamazepine E. Start treating the patient with an antidepressant such as paroxetine

Start treating the patient with an antidepressant such as paroxetine

A 29-year-old single woman is brought to the emergency department by the police after they picked her up attempting to break into a grocery store. When they apprehended her, they noticed that she "seemed high" and that she was sweating with dilated pupils. The patient admits to "doping" daily for the majority of the past year and losing 30 lb in the past 6 months. She claims that her habit now costs more than $100 per day, although she used to get the "same high" for $20. When intoxicated, she describes her mood as "really good" and that she has "loads" of energy. When she does not use, she craves the drug, becomes very sleepy, feels depressed, and has a large appetite. She has tried to quit on numerous occasions, even entering an inpatient treatment program at one point, but she always quickly begins using again. The patient used to work part-time as a secretary, but she lost her job because she was chronically late and, in fact, stole money in order to pay her dealer. She freely admits that she was trying to rob the grocery store to "pay off my debts." What is the most likely diagnosis?

Stimulant (cocaine) intoxication and stimulant (cocaine) use disorder

Which of the following approaches would be the most effective for the patient in Question 27.1 (13yo with seizures)? A) Confrontation about intentionally producing symptoms B) Explanation that the symptoms are not real C) Reassurance that a cause will be found D) Suggestion that symptoms will improve with time E) Suggestion that the family begins therapy

Suggestion that the symptoms will improve with time

A 23 year old woman with a diagnosis of histrionic personality disorder comes to see her primary care provider for the chief complaint of frequent headaches. As the (male) provider is taking the patient's history, he notices that the patient is frequently reaching across the desk to touch his arm as he talks to her, as well as leaning forward in her seat to be nearer to him. She expresses gratitude in his listening to her. Which of the following responses is the most appropriate response from the provider during this interview? A. Tell the patient to stop touching him immediately B. Move his seat further from the patient so that she cannot reach him C. Tell the patient that she will be referred to a female provider D. Tell the patient that he understands her concern about her headaches, but touching him is inappropriate E. Tell the patient he understands her gratitude in this situation

Tell the patient that he understands her concern about her headaches, but touching him is not appropriate

The patient in Questions 14.1 and 14.2 (persistent depressive disorder) returns to your office for a medication check. Sertraline was started 4 months ago. The dose has been increased twice, and the patient has been taking 200 mg for 2 months. She feels the medication has provided some, but not total, relief from her symptoms, and she has tolerated the medication well. You augment with 40 mg of duloxetine. The next morning the patient calls to report she is not feeling well. She has a headache, chills, her heart is racing, and her temperature is 102 °F (38.9 °C). What instructions should you give your patient? A Have her call her primary care provider B Have her come to your office this afternoon when you have an opening C Nothing; it is probably a virus D Tell the patient to go directly to the emergency department E Tell her to take ibuprofen to control the temperature and headache and report back in the morning

Tell the patient to go directly to the emergency department

Psychotic depression is diagnosed in a 14-year-old boy, and he is treated with an antipsychotic agent (risperidone) and an antidepressant. Three months later, his mood symptoms have resolved and he is no longer psychotic. Which of the following best describes the next step? A Both medications should be discontinued via a taper. B The antipsychotic medication should be discontinued via a taper. C The antidepressant medication should be discontinued via a taper D Both agents should be continued for 6 to 9 months. E The antipsychotic medication should be stopped immediately

The antipsychotic medication should be discontinued via a taper

Which of the following statements is accurate regarding the treatment of narcissistic personality disorder? A. Psychotropic medications are used to specifically treat NPD B. The mainstay of treatment is individual psychotherapy C. Patients with NPD may benefit from prolonged inpatient treatment D. Group therapy should be initiated in almost all patients with NPD

The mainstay of treatment is individual psychotherapy

The patient described in Question 32.1 (5ft 6in, 90lbs) was admitted to an inpatient pediatric unit for initial treatment. She received nutritional support provided to her by the treatment team, which included a dietician. However, the patient wanted to get better faster and started to take one additional 30-g protein shake per meal. On day 4 of treatment, the patient developed signs and symptoms of cardiac failure and a new arrhythmia. What is the most likely cause of this complication? A) Secretive purging has resulted in electrolyte disturbance B) Starting olanzapine has caused the arrhythmia, leading to cardiac failure C) The patient used an illicit substance in the hospital that caused the symptoms D) The patient developed refeeding syndrome E) The patient developed a deep vein thrombosis

The patient developed refeeding syndrome

A 48-year-old woman is brought to the emergency department. She is unresponsive to questions, stumbles around the room, and is agitated. On physical examination, you notice that she smells of alcohol, and she is not cooperative during the remainder of the examination. Administration of what medicine would be the most appropriate initial treatment? A) Antipsychotic B) Benzodiazepine C) Disulfiram D) Glucose E) Thiamine

Thiamine

A 19-year-old woman presents to your office for follow-up after an inpatient psychiatric hospitalization. While in the hospital, she was diagnosed with bipolar disorder and was started on lithium. She now has questions about side effects and monitoring. Which of the following should be monitored regularly while taking lithium? A Hemoglobin and hematocrit B Liver enzymes C Potassium D Thyroid function

Thyroid function

A 17-year-old adolescent girl is seen in your office after her friends noticed some strange behavior. The patient reports to you that in addition to some long-term depression issues, she has begun to experience some other disturbing events. She reports that over the last 2 months she has been hearing voices—both at work and at home—of people she does not think are there. She does not recognize these voices. Sometimes they just give a dialogue of what she is doing, but more disturbing to her is when they start saying horrible things about her and tell her to do things she does not want to do. You start her on olanzapine, and when she returns in 1 week, the voices have been gone entirely for 2 or 3 days. However, she continues to experience severe mood symptoms. Her Hamilton Depression Rating Scale score places her in the moderate to severe range for depression. Which of the following should you do next? A Inform the patient that these symptoms are negative symptoms common to the disorder. B Refer the patient for supportive psychotherapy. C Treat the patient with fluoxetine (an SSRI). D Increase the dose of the antipsychotic. E Add a mood stabilizer to the regimen.

Treat the patient with fluoxetine (an SSRI)

A 47-year-old man is admitted to a psychiatric unit for depression with suicidal ideation and detoxification. He has a long history of dependence upon both alcohol and cocaine. Which of the following signs is most characteristic of early alcohol withdrawal? A) Decreased blood pressure B) Hypersomnia C) Persistent hallucinations D) Tremor E) Increased appetite

Tremor

An adult must have symptoms of persistent depressive disorder for what length of time in order to meet DSM 5 criteria and receive a diagnosis? A. Two years B. Six months C. One year D. Two weeks

Two years

A 27-year-old woman presents to the clinic complaining of a depressed mood for the past two weeks. She states she has had significant weight gain, has been sleeping more than usual, has decreased interest in normally pleasurable activities, feels guilty, and has had decreased concentration. She states she had a hypomanic episode in the past while she was using amphetamines. Which of the following is the most likely diagnosis? A Bipolar disorder type 1 B Bipolar disorder type 2 C Persistent depressive disorder D Unipolar depression

Unipolar depression

A 29-year-old woman with no medical or psychiatric history is brought to the emergency department (ED) by her husband because of her bizarre behavior. She has been ranting about being a victim of gang stalking ever since she lost her job at the postal service. "I know they are after me," she states repeatedly. Her husband states that since being laid off, the patient has been spending much of her time "with the neighborhood druggies." Which of the following would the initial workup include? A Computed tomography (CT) scan of the head B Brain imaging with magnetic resonance imaging (MRI) C Urine drug screen D Spinal tap E Ceruloplasmin level

Urine drug screen

Which of the following is first-line treatment for a patient with acute severe mania? A Electroconvulsive therapy B Lamotrigine C Lithium D Valproate and olanzapine

Valproate and olanzapine

40-year-old man who has been stable on methadone maintenance for the past 10 years approaches you about switching to buprenorphine/naloxone because he is having difficulty attending the methadone clinic daily. He learned that you prescribe buprenorphine/naloxone in your clinic. He has done well on methadone without any relapses on heroin. He uses marijuana several times a week to manage his anxiety because his health care provider told him not to mix benzodiazepines with methadone. How would you counsel him about a potential switch to buprenorphine/naloxone? a. Tell him methadone is the best treatment for him because he has done so well over the past 10 years b.Work with the methadone provider to taper him off. Explain that he may experience withdrawal symptoms during the transition to buprenorphine/naloxone c.Inform him that his marijuana use makes him a poor candidate for buprenorphine/naloxone treatment d.Refer him to an inpatient substance abuse program for transition from methadone to buprenorphine/naloxone e. Let him know that buprenorphine/naloxone is much safer than methadone and he should have switched over a long time ago.

Work with the methadone provider to taper him off. Explain that he may experience withdrawal symptoms during the transition to buprenorphine/naloxone

What diagnosis should be given to a patient who has non bizarre delusions for at least a month and no other symptoms? A schizoaffective disorder B delusional disorder C brief psychotic disorder D schizophreniform disorder

delusional disorder

A 56-year-old divorced, unemployed man with a long-standing history of substance abuse presents to the emergency department with abdominal pain, sweats, diarrhea, and body aches. On initial evaluation, the patient is noted to have a watery nose and eyes, a slightly elevated temperature of 100 °F (37.8 °C), and dilated pupils. His mood is dysphoric, and his affect is irritable and labile. His abdominal examination is benign. Laboratory examinations including electrolytes, complete blood count, liver function tests, amylase, and lipase are all normal. A plain abdominal x-ray showed no clear cause for his abdominal pain. dx? Medications to help alleviate symptoms: ?

dx-Opioid withdrawal medication- Buprenorphine with or without naloxone

Which of the following drugs is the best choice in the treatment of schizophrenia? A. risperidone B. clozapine C. haloperidol D. lithium carbonate E. fluoxetine

risperidone

A 42-year-old woman is hospitalized for severe abdominal pain. After an extensive evaluation, the clinicians have reached a diagnosis of factitious disorder. Which of the following is most likely the motivation behind this patient's behavior? A) The motivation is unconscious, and thus the patient is unaware of it B) Desire to avoid jail C) Desire to take on the patient role D) Desire to obtain compensation E) Desire to obtain narcotics

Desire to take on the patient role

What is a common differential for dissociative disorders that must be ruled out before starting psychotherapy?

Dissociation related to substance abuse

23-year-old female is reported missing by her parents in California. Years later, it is discovered that she is living in New York. She has no recollection of her previous life in California.

Dissociative amnesia

A dissociative disorder characterized by the sudden and extensive inability to recall important personal information, usually of a traumatic or stressful nature.

Dissociative amnesia

A 43-year-old schoolteacher is surprised when a parent states that she saw him at a strip club several days ago. He denies these accusations and truly hasno memory of the event.

Dissociative identity disorder

A rare dissociative disorder in which a person exhibits two or more distinct and alternating personalities. Also called multiple personality disorder.

Dissociative identity disorder

A paraphilic disorder characterized by repeated and intense sexual urges, fantasies, or behaviors that involve being humiliated, beaten, bound, or otherwise made to suffer.

Sexual masochism disorder

The diagnosis of conduct disorder is made when which of the following criteria is (are) fulfilled? A) Repetitive and persistent patterns of behavior that violate the rights of others B) Stealing C) Lying D) Vandalism E) A and any two of B, C, and D

A and any two of B, C, and D

Which of the following presentations is characteristic of dissociative amnesia? A) A 29 year old nun is surprised when a man states that he saw her at an exotic dancing establishment several days ago. She denies these accusations and truly has no memory of the event B) A 39 year old man presents stating that several times in the last month he felt as if he was outside of his body. He struggles to describe the experience but says the feeling best approximates "watching a movie of himself" C) A 21 year old woman is reported missing by her parents in Miami. Years later it is discovered that she is living in Texas. She has no recollection of her previous life in Miami D) A 25 year old woman is brought to the emergency room by EMS when she was found naked in a clothing store. Her family and friends said that for the last 2 weeks she has been increasingly grandiose, had been sleeping 3-4 hours per night, and had not gone to work E) An 82 year old man is found several states away from his home and is not oriented to person, place, or time

A 21 year old woman is reported missing by her parents in Miami. Years later it is discovered that she is living in Texas. She has no recollection of her previous life in Miami

In the patient from Question 2.2 (ASD, spinning objects), what statement might best describe the etiology of the disorder? A) Measles, mumps and rubella (MMR) vaccination has been shown to be a major factor in the development of autism B) A cold, unyielding mother and absent father are significant factors leading to ASD C) A deficit in oxytocin leads to ASD D) A complex heritable and in utero environmental influence best explain the etiology of ASD E) Brain trauma at birth leads to ASD

A complex heritable and in utero environmental influence best explain the etiology of ASD

Which of the following is required for the diagnosis of gender dysphoria in children but not in adolescents or adults? A) A strong desire to be treated as the other gender B) A strong desire to be of the other gender or an insistence that one is the other gender C) A strong preference for playmates of the other gender D) A strong desire for the primary and/or secondary sex characteristics that match one's experienced gender E) A strong preference for cross-gender roles in make-believe or fantasy play

A strong desire to be of the other gender or an insistence that one is the other gender

A 17-year-old adolescent girl is brought to see a psychiatrist because her parents have become increasingly alarmed about her weight loss. The patient claims that her parents are "worrying about nothing" and that she has come to the office just to appease them. She states that she feels fine, although her mood is slightly depressed. She denies having problems with sleeping or appetite and denies any kind of drug or alcohol abuse. She says that she thinks she looks "fat," but that if she could lose another couple of pounds, she would be "just right." She notes that her only problem is that she stopped having her period 3 months ago; she is not sexually active and therefore cannot be pregnant. When questioned separately, the parents report that the patient has been steadily losing weight over the past 8 months. They say that she started dieting after one of her friends commented that she "looked a little plump." At that point, they noted that their daughter weighed approximately 120 lb. The patient lost 5 lb and, according to the parents, felt good about the comments made by her friends. Since that time, she has eaten less and less. She now dresses in baggy clothes and does not discuss with her parents how much she weighs. Despite this, she helps her mother cook elaborate meals for party guests when the family entertains. She exercises throughout the day, and her parents say they can often hear her doing jumping jacks and sit-ups in her room in the evening. On physical examination, the patient is found to be 5 ft 2 in tall; she weighs 70 lb and appears cachectic. A) What is the most likely diagnosis? B) What are the next therapeutic steps?

A) Anorexia nervosa, restrictive type B) The patient might benefit from hospitalization or day treatment, but it is unlikely that she will agree. However, because the patient is a minor, her guardian can sign her into a hospital without her consent. Her initial treatment should be aimed at restoring her nutritional status, as she is grossly malnourished. Dehydration, starvation, and electrolyte imbalances must be corrected. The patient should be weighed daily, and her daily fluid intake and output should be monitored. Therapy (behavioral management, individual psychotherapy, family education, and group therapy) should also be started, but her unstable nutritional status needs to be addressed first.

A 7-year-old girl is brought to her pediatrician on the suggestion of her second-grade teacher. The patient has been back in school for 3 weeks following a summer break. According to the teacher, the patient has found it very difficult to complete her classroom tasks since returning to school. The child is generally not disruptive but is unable to finish assignments in the allotted time, although her classmates do so without difficulty. She also makes careless mistakes in her work. Although she is still passing her classes, her grades have dropped, and she seems to daydream a great deal in class. The teacher reports that it takes several repetitions of the instructions for the patient to complete a task (eg, in an art class). The patient enjoys physical education and does well in that class. The child indicates that when it appears to others that she is not paying attention she is thinking about other things. Teachers report that her attention wanders constantly and they have to call her name or wave to get her immediate attention. There have been no episodes where she stares blankly or is briefly nonresponsive. Although her parents have noticed some of the same behaviors at home, they have not been particularly concerned because they have found ways to work around them. If they monitor the child and her work directly, she can complete her homework, but they must continually check her work for careless mistakes. She does seem to know the right answer when it is pointed out. The parents also report that the patient does not get ready for school in the mornings without moment-by-moment monitoring. Her bedroom is in shambles, and she loses things all the time. The parents describe their daughter as a happy child who enjoys playing with her siblings and friends. They note that she does not like school, except for the physical education classes. A) What is the most likely diagnosis B) What are the recommended treatments for this disorder?

A) Attention-deficit/hyperactivity disorder (ADHD), predominantly inattentive presentation B) Use of a psychostimulant or atomoxetine along with behavioral parenting training and classroom behavior modification programs

A 2½-year-old boy is brought to a pediatrician by his parents for his regular yearly examination. He is the couple's only child. The parents relate a normal medical history with a single episode of otitis media. They recently placed their son in day care for 2 half-days a week. However, he has not adjusted well, crying and having tantrums during the first hour of day care. Then he usually quiets down, but he does not interact with the rest of the children. The teacher cannot seem to make him follow directions and notes that he does not look at her when she is near him and attempting to interact with him. On further discussion with the parents, the pediatrician finds that the patient has a limited vocabulary of perhaps 10 words. He does not use these words in any greater length than two words in a row and often uses them inappropriately. He did not speak his first clear word until 6 to 9 months. The patient does not interact well with other children but does not seem upset by them either. His favorite toys are often used inappropriately—he performs single, repetitive movements with them for what seems like hours on end. The pediatrician picks the child up to help him onto the examination table and notices that he seems quite stiff, pushing himself away from the examiner with his hands. Although his hearing and eyesight appear to be intact, the child does not respond to requests by the pediatrician and does not make eye contact. All other gross neurologic and physical features are within normal limits. A) What is the most likely diagnosis? B) What is the most likely prognosis for this condition?

A) Autism spectrum disorder B) The child will likely experience a number of developmental delays, but with intensive treatment at home and at school, he could achieve near-normal or normal development. Language development is the most important indicator of future developmental potential in ASD children

A 19-year-old woman is referred to a psychiatrist after her roommates become concerned about her behavior. The patient tells the psychiatrist that for the past 2 years, since beginning college, she has been making herself vomit by sticking her fingers down her throat. This behavior occurs regularly, as many as three or four times a week, and worsens when she is stressed out at school. The patient says that she regularly gorges herself with food and is worried that she will become overweight if she does not vomit it up. She describes her gorging episodes as "eating whatever I can find" in large quantities and mentions one incident in which she ordered three large pizzas and ate them all by herself. The patient states that she feels out of control when she is gorging herself but is unable to stop. She is ashamed of this behavior and goes to great lengths to hide how much she eats. She notes that her sense of self-esteem seems to be very dependent on her weight and whether or not she sees her body image as fat. She agreed to see a psychiatrist after her roommates found out about the self-induced vomiting. A physical examination shows a young woman, 5 ft 6 in tall and weighing 135 lb. Her vital signs are: blood pressure 110/65 mm Hg, respirations 12 breaths per minute, temperature 98.2 °F (36.8 °C), and pulse rate 72 beats per minute (bpm). The rest of her physical examination is within normal limits. A) What is the most likely diagnosis for this patient? B) What treatment modalities should the psychiatrist recommend? C) What areas of the physical and laboratory evaluation should receive special attention?

A) Bulimia nervosa B) Nutritional rehabilitation, cognitive behavioral psychotherapy, and treatment with an antidepressant (selective serotonin reuptake inhibitor [SSRI]) C) The physical should include parotid glands, mouth, teeth for caries, abdominal examination for esophageal or gastric injury, and assessment for dehydration from laxative use, ipecac-associated hypotension, tachycardia, and arrhythmias. Serum electrolytes, magnesium, and amylase levels should be checked

A 15-year-old girl is brought to a psychiatrist by her parents because they are concerned that she might be depressed. The parents had no complaints until 2 or 3 years ago. Since then, the patient's grades have fallen because she cuts classes. She gets into fights, and her parents claim that she hangs out with the "wrong crowd"; some nights she does not come home until well past her curfew. The patient says that there is "nothing wrong" with her and that she wants her parents to "butt out of [her] life." She claims that she is sleeping and eating well. She says she skips school to hang out with her friends and admits that they frequently steal food from a convenience store and spend time watching movies at one of their homes. She claims that she fights only to prove that she is as tough as her friends but admits that she often picks on younger students. She shows limited guilt or remorse about this. She seems cold and uncaring about how her behavior might affect others. She is not concerned about her grades and just wants her parents to "lay off" and let her enjoy her youth. She denies the use of drugs or alcohol other than occasionally at parties. Her blood alcohol level is zero, and the results of a urinalysis are negative for drugs of abuse. A) What is the most likely diagnosis? B) What treatment should be started?

A) Conduct disorder (CD) B) A multisystemic treatment (MST) approach with involvement of parents and teachers. The treatment of CD can be difficult. There are only a few studies that look at the treatment of CD systematically. Over the last few years, a number of new studies have begun to investigate how to treat CD. In terms of behavioral interventions, multisystemic therapeutic approaches are quite helpful. These approaches combine a well-coordinated plan to help parents develop new skills at home, such as parent-child interaction training, to help the relationship between parents/caregivers and the child. In addition, it is helpful to teach classroom social skills, institute playground behavior programs, and facilitate and encourage communication between teachers and parents. Psychopharmacologic interventions also show some promise. Behavioral and cognitive behavioral group−based parenting interventions work well and are cost-effective for improving child conduct problems, parental mental health, and parenting skills in the short term.

A 24 year old man is admitted to the neurology service with new-onset blindness. The patient awoke on the morning of his admission entirely unable to see. A detailed workup by the neurology service, including physical, laboratory studies, and imaging, revealed no clear medical reason for this abnormality -- the patient was found to be otherwise healthy. A psychiatric consultation was subsequently ordered.The patient tells the psychiatrist that he does not know why he is blind. He emigrated from Mexico several years ago, coming to the United States to make money to support his sick mother. She remained ill for several years, but he was unable to send her money because he lost the money gambling. She died recently, and he became despondent because he would never see her again. On mental status exam, the patient is alert and oriented to person, place, and time. His appearance and hygiene are good, and he does not seem to be overly concerned with his blindness, His mood is described as "okay," and his affect is congruent and full rate. He has normal thought processes and denies having suicidal or homicidal ideation, delusions, or hallucinations. A) What is the most likely diagnosis for this patient? B) What is the most appropriate treatment for this patient?

A) Conversion disorder (functional neurologic symptom disorder) B) Educating the patient about the illness often results in resolution of symptoms, CBT and/or PT can also be useful

A 22 year old woman is brought to the clinic by her brother, who voices concern about the patient's strange behavior. He states that his sister struggles with depression and that it acutely worsened after a painful breakup with her abusive boyfriend. She did not return home for several days, after which he finally found her at a bar across town. She had facial bruising and appeared disheveled. His sister acted like a "completely different person" and spoke with a French accent. He was unaware of his sister ever learning French. He finally convinced her to come back home with him that night. Since returning home, the woman has "been her normal self." The brother admits that they had a rough childhood, often suffering from physical abuse at the hands of their father. He states that despite an unstable upbringing, "We have done pretty well for ourselves. Both of us are clean and employed."The patient denies any recollection of the past few days. Her last memory before the episode was her boyfriend yelling at her and his fist flying at her face. She felt herself disconnect from her body and cannot recall what happened next. She states that her boyfriend has been increasingly jealous and possessive of her. He accused her of cheating on him, which she adamantly denied. The patient reports that her boyfriend became abusive over the past few months when she began receiving late night phone calls from strange men asking "for the French girl" to "meet up again." She denies straying in her relationship and feels bewildered about these phone calls. The patient does not remember much of her childhood and has "blocked out" the abuse. On memory testing, the patient is oriented to person, place, and time and does not exhibit any gross impairment. A)What is the most likely diagnosis for this patient? B) What could be a causal factor and is highly associated with this disorder?

A) Dissociative Identity Disorder (DID) B) Most DID patients have experienced significant physical and sexual trauma, often during their childhood. Creation of an alternate identity can be conceptualized as a protective but maladaptive mechanism for the core personality

A 10-year-old boy with chronic asthma is brought to a pediatrician for his 6-month checkup. He has complained of chronic headaches for the past 3 months, as well as increasing gastric upsets, which his family believes are caused by multiple food allergies. The patient has a severe allergy to peanuts, which limits the number of places he can go in public. Thus, he has been home schooled for a year and is doing well. A review of his history shows that he is a highly articulate, thoughtful child who appears to be at or above the educational level of his peers. The child does not agree to be interviewed separately from his mother, stating, "I don't go anywhere without my mother." The two of them are almost never apart. Two years ago, the mother was hospitalized after a serious bout with lupus. She continues to struggle with her disease, and despite having a thriving career before her illness, she can do very little now. She is home all the time, dealing with her own recovery and the management of her illness. During her hospitalization, the patient was quite worried about her illness and even now believes that if he is not around to monitor her condition, she might get sick and require hospitalization again—or even worse. The mother has difficulty sleeping and is most comfortable on the living room couch. The patient no longer uses his own room but sleeps in a chair next to his mother to continue to keep an eye on her. He has very few friends and can be separated from his mother only briefly, and only if he is in the company of his brother or father. After a short period, he becomes anxious and upset and must be reunited with his mother. A) What is the most likely diagnosis? B) What is the prognosis for this disorder? C) What treatments might be helpful in this disorder?

A) Separation anxiety disorder B) This condition is difficult to treat; without treatment, the patient is likely to develop depression and/or an anxiety disorder as an adult C) A multisystemic treatment approach is required. Selective serotonin reuptake inhibitors (SSRIs) can be helpful in the management of mood symptoms and anxiety. Relaxation techniques can help, along with a gradual separation program. Home schooling only reinforces the child's fear of separating from the family and should be reconsidered

A 41-year-old nurse presents to the emergency department with concerns that she has hypoglycemia from an insulinoma. She reports repeated episodes of headache, sweating, tremor, and palpitations. She denies any past medical problems and only takes nonsteroidal anti-inflammatory medications for menstrual cramps. On physical examination, she is a well-dressed woman who is intelligent, polite, and cooperative. Her vital signs are stable except for slight tachycardia. The examination is remarkable for diaphoresis, tachycardia, and numerous scars on her abdomen, as well as needle marks on her arms. When asked about this, she says that she feels confused because of her hypoglycemia.The patient is subsequently admitted to the medical service. Laboratory evaluations demonstrate a decreased fasting blood sugar level and an increased insulin level, but a decreased level of plasma C-peptide, which indicates exogenous insulin injection. When she is confronted with this information, she quickly becomes angry, claims the hospital staff is incompetent, and requests that she be discharged against medical advice. A) What is the most likely diagnosis? B) How should you best approach this patient?

A) Factitious Disorder B) In order to engage the patient in psychiatric treatment, attempt to ally with her. Working in conjunction with the patient's primary care provider is often more effective than working with the patient alone. Focus on patient management versus curing the patient. Personal awareness of one's own feelings toward the patient must be maintained, as it is often very easy to get angry with such patients and behave punitively

A 32-year-old man and his 28-year-old wife come to a psychiatrist because of problems in their relationship. The wife states that the two have been married for 6 months and that they dated for 2 months prior to that. During all their sexual encounters, the husband insists that the wife wear very high-heeled shoes at all times. Although the wife initially thought that this behavior was sensual, she now worries that it is the shoes that the husband finds attractive and not her. She thinks the behavior is "freaky" and has asked the husband to stop, which he has refused to do. This has led to arguments between them. The husband states that he is unable to achieve an erection or orgasm without the presence of the shoes. He notes that for as long as he can remember, he has needed high-heeled shoes as part of his sexual play. He feels no shame or guilt about this behavior, although he is worried that it is causing problems between him and his wife. A) What is the most likely diagnosis for the husband? B) What is the course of and prognosis for this disorder?

A) Fetishistic disorder B) The course of this disorder is chronic, and it has a poor prognosis

A 14-year-old girl presents to the psychiatrist for a new patient evaluation. She endorses depression, anxiety, and feeling like "there is a pit in my stomach and things are spiraling downward." She states she has a lot she wants to do with her life and does not think she will achieve it. She is a straight-A student and enjoys band and art. She states she has been crying a lot and sometimes it feels as if "I do not exist." She describes herself as a tomboy and states she has always been interested in sports. She states she feels more comfortable in boy's clothing. She states she is uncomfortable with her body, especially the development of her breasts. She feels she should have been born a male. She has felt this way since she was 10 years old but was fearful of telling her family due to their strong religious beliefs. She wishes to wear a binder for her breasts and wants to start hormone therapy. She was attracted to both males and females initially and believed she was bisexual. She now reports an exclusive interest in girls. She states she did some research because she didn't feel she identified with being bisexual or lesbian and now identifies as transgender. She recently disclosed to her family 2 months ago that she identified as a male and states this is stressful because her parents do not understand. She states she prefers the use of male pronouns with regard to herself, and she has picked a male name. She is afraid to disclose these facts to her family.On mental status exam, she is well-groomed and cooperative. She is dressed in boy's clothing brands consisting of jeans and a t-shirt with a button-down shirt over it. Her hair is shortly cropped, and she is wearing a baseball cap. She is alert and oriented to person, place, and time. No abnormalities were found on her mental status examination. A) What is the most likely diagnosis? B) What options are open to the patient other than sex reassignment surgery?

A) Gender dysphoria B) It is recommended that fully reversible steps precede partially reversible procedures, which should precede irreversible procedures. Fully reversible steps include living as the desired gender and pubertal suppression. Partially reversible procedures include administration of gonadal hormones to bring out desired secondary sex hormone characteristics. Irreversible procedures include gender-affirming surgeries.

A 42-year-old woman presents to her primary care provider with a chief complaint of back pain for the past 6 months that began after she was knocked down by a man attempting to elude the police. She states that she has extreme pain on the right side of her lower back, near L4 and L5. The pain does not radiate, and nothing makes it better or worse. She says that since the injury she has been unable to function and spends most of her days lying in bed or sitting up, immobile, in a chair. Immediately after the accident, she was taken to an emergency department, where a workup revealed back strain but no fractures. Since then, the patient has repeatedly sought help from a variety of specialists, but the ongoing pain has been neither adequately explained nor relieved. She denies other medical problems, although she mentions a past history of domestic violence that resulted in several visits to the emergency department for treatment of bruises and lacerations.On mental status examination, the patient is alert and oriented to person, place, and time. She is cooperative and maintains good eye contact. She holds herself absolutely still, sitting rigidly in her chair and grimacing when she has to move even the smallest amount. Her mood is depressed, and her affect is congruent. Her thought processes are logical, and her thought content is negative for suicidal or homicidal ideation, delusions, or hallucinations. A) What is the most likely diagnosis of this patient? B) What is the best approach to this patient?

A) Somatic symptom disorder with predominant pain B) Validate the patient's experience of pain. Explain the role of psychological factors as a cause and consequence of pain. Consider antidepressants, therapy, alternative therapies such as yoga, and referral to a pain clinic

Conduct disorder appears to result from an interaction of which of the following factors? A) Temperament B) Attention to problem behavior and ignoring of good behavior C) Association with a delinquent peer group D) A and C E) A, B, and C

A, B, and C

Which of the following laboratory abnormalities would most likely be found in the patient in Question 31.2 (binging and purging)? A) Hypermagenesemia B) Low serum amylase level C) Hypochloremic-hypokalemic alkalosis D) Elevated thyroid indices E) Hypercholesterolemia

Hypochloremic-hypokalemic alkalosis

A 12-month-old male whose mother is concerned that he does not seem to play with other children as his brother and sister did at this age. She indicates she has noticed that he does not seem to respond when she or other children call him by name, he is indifferent to other children or adults when they are present, and he does not seem to know any words and "just grunts."

Autism spectrum disorder

A 9-year-old boy is referred to a psychiatrist because of poor school performance. He has been tested for learning disabilities but none are present, and he has an IQ in the high normal range. The teacher reports that it is hard to hold his attention and that he appears hyperactive and fidgety at school, which disrupts the class. However, he does not purposefully go out of his way to disobey the teacher. His parents have noticed no difficulties at home, but his soccer coach has noticed attention problems during practice, and his Sunday school teacher has trouble teaching him because of distractibility. Which of the following is the most likely diagnosis for this patient? A) ADHD, combined type B) ADHD, predominantly hyperactive type C) ADHD, predominantly inattentive type D) Oppositional defiant disorder E) No diagnosis, because the ADHD symptoms must be reported in the home

ADHD, combined type

Which of the following disorders often appear together in the same individual at various life stages? A) Mental retardation, attention-deficit/hyperactivity disorder (ADHD), and learning disability B) Childhood depression, ADHD, and early-onset adult schizophrenia C) ADHD, conduct disorder, and antisocial personality disorder D) Adjustment disorder, ADHD, and major depression E) ADHD, bipolar disorder, and conduct disorder

ADHD, conduct disorder, and antisocial personality disorder

Which treatment(s) is (are) used to manage symptoms of conduct disorder? A) Behavioral therapy B) Mood stabilizers C) Alpha agonists D) Beta blockers E) All of the above

All of the above

A 16-year-old adolescent girl is brought into the office due to having suicidal thoughts. The parents report that over the past year, the patient has increasingly been truant from school. She has stolen money from her family and neighbors, and she slashed the tires of a fellow classmate for making fun of her. She often bullies her sister and has slapped her on several occasions. Over the past 2 months, the patient has had increasing problems sleeping and has lost weight. Upon questioning, the patient states that she has low energy. A urine drug screen is negative. Which of the following treatments is best employed to treat this patient? A) MST B) Attendance in group therapy C) An antidepressant medication D) Treatment of the family to address the underlying reasons for the depression E) Helping the adolescent

An antidepressant medication

A 55-year-old man complains of inability to achieve an erection. He has been worried about his health recently and takes antihypertensive medication. Which of the following would most likely differentiate between an organic and psychiatric condition? A) A lower-extremity myographic examination B) Magnetic resonance imaging of the lumbosacral spine C) An erection on awakening in the morning D) The interpretation of projective tests E) An electroencephalographic reading

An erection on wakening in the morning

Atomoxetine is a relatively new drug used for the treatment of ADHD. Which of the following represents the advantage of using atomoxetine over methylphenidate? A) Atomoxetine has a shorter half-life B) Atomoxetine is available in a generic form that is less costly than methylphenidate (Ritalin) C) Atomoxetine appears to have less of a potential for abuse than does methylphenidate (Ritalin) D) Atomoxetine's effects begin working immediately to reduce symptoms of ADHD E) Atomoxetine can be taken on an empty stomach

Atomoxetine appears to have less of a potential for abuse than does methylphenidate (Ritalin)

A 9-year-old boy is brought by his primary care physician by his mother after the child was suspended from school for being disruptive during class. The teacher noted that the patient is always fidgeting, has difficulty concentrating, and does not complete assignments. His mother notes that he often gets up and runs around the house when she is trying to call everyone for dinner and that this has been going on since he was 6 years old.

Attention-deficit/hyperactivity disorder

Which of the following treatment options would be a contraindicated treatment option for the patient in Questions 31.2 and 31.3 (bulimia, binging and purging)? A) Nutritional rehabilitation B) Cognitive behavioral psychotherapy C) Careful use of SSRIs D) Group psychotherapy E) Atypical antipsychotic medications

Atypical antipsychotic medications

In controlled studies, which of the following medications is ineffective for the use in childhood anxiety disorders? A) Venlafaxine B) SSRIs C) Buspirone D) Tricyclic antidepressants E) Benzodiazepines

Benzodiazepines

The patient in Questions 29.1 and 29.2 (grandmother with somatic symptom disorder) feels that her headaches are now unbearable. Which of the following treatments is the most appropriate? A) Acetaminophen B) Biofeedback C) Lorazepam D) Non-steroidal anti-inflammatory medication E) Oxycodone

Biofeedback

A 26-year-old woman presents to her provider with the chief complaint of "I have epilepsy." She states that for the past 3 weeks she has had seizures almost daily. She describes the episodes as falling on the ground, followed by her arms and legs shaking uncontrollably. These events last for approximately 10 minutes. She is unable to otherwise move during the time, although she denies any loss of consciousness or bladder or bowel functions. The patient seems to be rather indifferent when stating her complaints. She has never injured herself during these episodes, but as a result, she has been unable to continue her job. She is somewhat bothered because she received a promotion 1 month ago. Which of the following is the most likely diagnosis? A) Body dysmorphic disorder B) Conversion disorder C) Illness anxiety disorder D) Seizure disorder E) Somatic symptom disorder

Conversion disorder

A 13-year-old boy is brought in by police officers to the emergency department, where his parents soon arrive. He was placed under arrest and charged with destruction of property as well as assault and battery after police arrived at the scene and found the patient beating the owner of a liquor store with a baseball bat. According to police, the patient cut his arm on a piece of glass while jumping through a store window in an attempt to evade arrest. The police officer tells you, "We know this one ... he's a regular customer of ours. We've had him down at juvenile hall more times than I can count." The patient's parents tell you that his aggressive behavior started in the past couple of years: "Sure, he would argue and fight with us when he was younger, but he didn't start getting arrested until he was about eleven." His parents recount a number of disturbing instances of cruelty to animals, leading to the death or maiming of several neighborhood cats. After an examination, you note that his wounds are not serious. His vital signs are within normal limits. The patient appears cheerful and unconcerned. He is worried about getting the blood off of his favorite baseball bat more than about the fate of the man he injured.

Conduct disorder

A 16-year-old girl is brought to her primary care provider by her mother, who states that her daughter has been losing weight steadily. The adolescent denies there is a problem and states that she is in no way underweight. The provider determines that the girl is 5 ft 6 in tall and weighs 90 lb. Which of the following would be the next best step to work this patient up? A) CBC and differential white blood cell count B) Thyroid function studies C) Serum potassium level D) Calculation and assessment of patient's BMI E) Liver function studies

Calculation and assessment of patient's BMI

Thought to be the main risk factor for dissociative identity disorder.

Childhood abuse

Which of the following options is the first-line treatment for the patient in Question 31.2 to 31.4 (bulimia, binging and purging)? A) Nutritional rehabilitation B) Cognitive behavioral psychotherapy C) Careful use of SSRIs D) Interpersonal therapy E) Topiramate

Cognitive behavioral psychotherapy

A 23-year-old pregnant woman complains of an inability to feel her legs. She wonders if the fetus is grabbing her spinal cord. Although she does not appear concerned about her condition, on further questioning she admits that her pregnancy was unplanned and that it has been a source of stress for her and her husband. Her neurologic examination is unremarkable except for decreased sensation below her waist. The results of a computed tomography scan and magnetic resonance imaging of her brain and spine are normal. Which of the following best describes the situation? A) Factitious disorder B) Malingering C) Conversion disorder D) Somatic symptom disorder E) Illness anxiety disorder

Conversion disorder

A 29-year-old man presents to his primary care provider with back pain for the past year since he slipped on a wet floor at work. He has been unable to work since the accident. A comprehensive medical workup has been entirely normal, and his only symptom is pain in the middle of his back that is present constantly. He has had multiple visits (more than two dozen) to the emergency department, where he is frequently given opioids and alprazolam to help with the pain. At today's visit, he tells you he is applying for disability designation and benefits. If the patient truly has pain and limitations without a medical physiologic explanation, which of the following is most likely to be present? A) Somatic symptom disorder B) Malingering C) Conversion disorder D) Posttraumatic stress disorder

Conversion disorder

A 13 year old girl is admitted to the pediatric unit for the evaluation of seizures. Her first seizure occurred 1 week ago, and she has had three since then. Episodes are described by her parents as a generalized shaking and jerking of the body, lasting 1 to 2 minutes, with eyes rolled back. During one seizure, there was urinary incontinence. The girl denies feeling depressed but does mention feeling "stressed out" due to school and recently not getting along with her stepmother. There is a history of sexual abuse by her stepfather 5 years ago. She has diagnoses of depressive disorder and anxiety for which she takes fluoxetine 20mg daily. Her physical examination and neurologic workup are unremarkable. Laboratory studies are also within normal limits. Which of the following is the most likely diagnosis? A) Body dysmorphic disorder B) Conversion disorder (functional neurologic symptoms disorder) C) Factitious disorder D) Illness anxiety disorder E) Malingering

Conversion disorder (functional neurologic symptoms disorder)

A 34-year-old female who reports that several times last month she felt as if she was outside of her body. She struggles to describe the experience but says the feeling best approximates "watching a movie of herself."

Depersonalization/derealization disorder. Depersonalization disorderoften starts manifesting in early adulthood, and is usually preceded by a traumatic event

A paraphilic disorder consisting of recurrent and intense sexual urges, fantasies, or behaviors that involve the use of a nonliving object or nongenital part, often to the exclusion of all other stimuli, accompanied by clinically significant distress or impairment

Fetishistic disorder

A 34-year-old woman presents with a 10-year history of episodes in which she eats large quantities of food, such as eight hamburgers and three quarts of ice cream, at a single sitting. Because of her intense feelings of guilt, she then repeatedly induces vomiting. This cycle repeats itself several times a week. She is extremely ashamed of her behavior but says, "I can't stop doing it." On examination, which of the following physical findings is most likely to be seen? A) Dental caries B) Lanugo C) Muscle wasting D) Alopecia E) Body weight at less than the 10th percentile of normal

Dental caries

A 22-year-old woman is referred for neuropsychological testing after reporting confusion and amnestic episodes. She has received bills in the mail for credit cards she does not remember opening. The woman has found clothing in her closet that she does not recall purchasing. She has been tagged in numerous pictures on Facebook with people she does not know and has been checked into venues unfamiliar to her. What is the most likely diagnosis? A) BPD B) DID C) Factitious Disorder D) Malingering E) PTSD

DID

A 38-year-old barista presents to her primary care provider with concern about memory problems. She has had trouble recalling significant personal information and life events. The patient reports that dating has been problematic because during intimate encounters, "I just disconnect. I feel so scared and ashamed. I'm floating above my body, looking down on what's happening." Her current boyfriend has described that sometimes she talks like a "little girl" and other times like a "sophisticated southern belle." He finds it endearing, but news of this behavior frightened the patient, as she has no recollection of acting this way. Her boyfriend has said he loves the way she plays piano, but she cannot remember ever touching the instrument. She drinks socially on the weekends but denies heavy use, blackouts, or withdrawal symptoms. Which of the following is the most likely diagnosis? A) Alcohol Use Disorder B) BPD C) Dissociative amnesia' D) DID E) PTSD

DID

A 35-year-old woman has been hospitalized repeatedly for severe diarrhea and dehydration. Multiple extensive evaluations including culture for infectious etiologies, stool analysis, upper and lower gastrointestinal endoscopy, and biopsies have returned negative. During her current hospitalization, the attending physician orders a lab analysis, which reveals the presence of laxative use. Which of the following is the most appropriate next step in approaching this patient? A) Confronting her with the feigning of symptoms B) Discharging her from the hospital without follow-up'C) Establishing a therapeutic alliance D) Pharmacotherapy E) Referring them to legal authorities

Establishing a therapeutic alliance

A 23-year-old male who goes to city park during the summer months in an overcoat. He enjoys walking around the park exposing his genitals to strangers. He then runs away so as to avoid getting caught.

Exhibitionistic disorder

Sexual gratification attained by exposing genitals to unsuspecting strangers

Exhibitionistic disorder

A 32 year old woman is admitted for second- and third-degree burns of her right hand, which she attributes to accidentally spilling hot oil while she was cooking dinner. Upon evaluation, the surgeon recognizes the patient as someone he treated for a similar burn on the same hand 3 months ago. Further detailed review of her medical records reveals that this is her sixth burn-related injury in 2 years. Upon further questioning, there does not seem to be an attached secondary gain. Which of the following is the most likely diagnosis? A) Body dysmorphic disorder B) Conversion disorder (functional neurological symptom disorder) C) Factitious disorder D) Illness anxiety disorder E) Malingering

Factitious disorder

A 38-year-old woman comes in for evaluation of an abscess on her thigh. Her chart documents frequent outpatient and hospital visits. She is admitted, her abscess is drained, and she is treated with antibiotics. Culture studies demonstrate microorganisms consistent with fecal matter, and a further physical examination reveals many old scars, presumably self-inflicted. Which of the following best describes the situation? A) Factitious disorder B) Malingering C) Conversion disorder D) Somatic symptom disorder E) Illness anxiety disorder

Factitious disorder

A 12-year-old girl is hospitalized with severe abdominal pain that has not responded to numerous medications. The comprehensive evaluation, including blood tests and imaging, has been normal. The patient's mother was noted to be very cooperative with medical personnel, but when a psychiatric evaluation was requested, the mother was very unhappy and stopped the consultation after 5 minutes. Investigation revealed that the patient and her mother had been seen in various emergency departments around the county over the past several years. After separating the mother from the patient for several hours, the patient began to improve. Which of the following is the most likely diagnosis? A) Body dysmorphic disorder B) Conversion disorder C) Factitious disorder imposed on another (Munchausen by proxy) D) Drug withdrawal

Factitious disorder imposed on another (Munchausen by proxy)

Despite her protest, the adolescent in the vignettes in Questions 32.1 and 32.2 (5ft 6in, 90lbs) is diagnosed with anorexia. After stabilization of her nutritional status on a specialized inpatient unit, she is discharged home with plans for follow-up therapy as an outpatient. Which of the following treatments have been shown to be effective in treating anorexia nervosa as an outpatient? A) Neuroleptic therapy B) Family-based treatment C) Brief supportive therapy D) Group therapy E) Insight-oriented psychotherapy

Family-based treatment

A 33-year-old woman with a chief complaint of trouble having intercourse. She is completely disinterested in sex and she is not receptive to her partner's attempts to initiate foreplay. She reports that when they do have sex she never achieves orgasm. This is affecting her current relationship with her boyfriend.

Female sexual interest/arousal disorder

A female dysfunction marked by a persistent reduction or lack of interest in sex and low sexual activity, as well as, in some cases, limited excitement and few sexual sensations during sexual activity

Female sexual interest/arousal disorder

A female brings her 29-year-old boyfriend to a couples therapist because she is uncomfortable with his behavior. She found him clutching her feet during intercourse, and noticed that he insists on being able to see her feet while they engage in sexual acts.

Fetishistic disorder

A 36-year-old man presents to his PCP for evaluation of his complaints. He is convinced that he has colon cancer despite being told that it is unlikely because of his young age. He occasionally notices traces of red blood on the toilet paper, which he had previously attributed to hemorrhoids, and abdominal cramps when he eats too much. A review of the records demonstrates numerous prior appointments in connection with the same or similar complaints, including repeatedly negative results from tests for occult fecal blood and normal results from colonoscopies. He continues to be worried about dying of cancer and requests another colonoscopy. Which of the following is the most likely diagnosis? A) Body dysmorphic disorder B) Illness anxiety disorder C) Somatic symptom disorder with predominant pain D) Somatic symptom disorder E) Conversion disorder

Illness anxiety disorder

A 42 year old man returns to his internist for the fourth time in 5 months with the same complaints of intermittent numbness of his fingers and indigestion. Although his medical workup has been unremarkable, this has failed to reassure him. He remains anxious and is now concerned that he has Celiac disease and requests a GI consultation. Which of the following is the most likely diagnosis? A) Body dysmorphic disorder B) Conversion disorder (functional neurological symptom disorder) C) Factitious disorder D) Illness anxiety disorder E) Malingering

Illness anxiety disorder

A healthy 25-year-old man becomes preoccupied with his health after his father has a stroke. This man monitors his blood pressure several times a day, starts an aggressive cardiovascular exercise program, and becomes a vegan. He spends 2 to 3 hours per day reading the latest research in strokes and other cardiovascular disease. He attempts to make quarterly appointments with his primary care provider even though there is no apparent reason to do so. Which of the following best describes the situation? A) Factitious disorder B) Malingering C) Conversion disorder D) Somatic symptom disorder E) Illness anxiety disorder

Illness anxiety disorder

Children or adolescents with separation anxiety disorder are at higher risk for which other psychiatric disorder? A) Malingering B) Somatization disorder C) Bipolar disorder D) Learning disability E) Major depression

Major depressions

A 12-year-old girl with a history of complex trauma is admitted to the inpatient child psychiatry unit for the management of disruptive behaviors. On the unit, she has a verbal altercation with a staff member. She stops responding directly to the staff member and appears very frightened. She turns away and begins wailing, "Don't hurt me, daddy. Please, please don't..." If the patient is forcibly placed into physical restraints, what would be the most likely result? A) Mortality B) Psychotic break C) Retraumatization D) Symptom resolution

Retraumatization

A 13-year-old boy presents to a psychiatrist for the first time and gives a history indicative of gender dysphoria. He states that he wishes to undergo treatment to suspend puberty. Which of the following treatment steps is most appropriate? A) Screening the patient for psychopathology B) Treating the patient with an antidepressant before surgery C) Treating the patient with hormones D) Real-life experience in the community E) Ongoing supportive psychotherapy

Screening the patient for psychopathology

A 53-year-old male complaining of a lack of desire for sex with his wife causing him marked distress for the past year. His wife has been very hurt by his lack of response to her advances and he reports that this is having a significant strain on their relationship. Prior to this, he was interested in sex and he and his wife would have intercourse 1-2x per month. He is very active and continues to compete in triathlons. He has no significant past medical history.

Male hypoactive sexual desire disorder

A male dysfunction marked by a persistent reduction or lack of interest in sex and hence a low level of sexual activity

Male hypoactive sexual desire disorder

A 34-year-old woman is admitted to the inpatient psychiatric unit after being brought to the emergency department by police because of an episode of aggression in which she destroyed her brother's house during an argument. She initially states she has no recollection of the incident. She then goes on to provide vague details about how she ended up at her brother's and discloses that she was told she did 5000 dollars' worth of property damage. She verbalizes repeatedly that she is concerned about the legal consequences of her actions. She reports she has never had an episode like this but doesn't appear particularly distressed about it. She refuses to allow consent for collateral information to be gathered. What is the most likely diagnosis? A) BPD B) DID C) Factitious disorder D) Malingering E) PTSD

Malingering

A 45-year-old man complains of lower back pain and weakness in his legs after lifting heavy boxes while at work. He says that he has not been able to go to work for several days. He requests treatment and a letter excusing him from work. On examination, he is found to have significant lumbar pain without spasms. The strength in his legs is decreased because of a lack of effort. His reflexes are within normal limits. He was observed to be ambulating without any problem in the waiting area prior to the appointment. Which of the following best describes the situation? A) Factitious disorder B) Malingering C) Conversion disorder D) Somatic symptom disorder E) Illness anxiety disorder

Malingering

A 50-year-old man is referred to a clinician because he has ongoing migraine headaches. His headaches are chronic and bilateral, are worse with loud noises and light, and occur without aura or vomiting. His physical examination is unremarkable except that the patient does not appear to be in significant distress. When he is presented with various options for treatment, including nonsteroidal anti-inflammatory medications, he becomes angry, demanding that acetaminophen with codeine is "the only thing that has ever helped" him. When he is told that non-narcotic medications should be tried first, he accuses the provider of not believing him and storms out of the clinic. Which of the following best describes the situation? A) Factitious disorder B) Malingering C) Conversion disorder D) Somatic symptom disorder E) Illness anxiety disorder

Malingering

Which evidence-based treatment is likely most effective for the treatment of CD? A) Parent training B) IOP C) Pharmacotherapy D) Military school E) Multimodal interventions

Multimodal interventions

Which of the following has the highest rate of comorbidity with ADHD? A) Learning disorder B) Anxiety disorder C) Mood disorder D) Oppositional defiant disorder/conduct disorder E) Tourette disorder

Oppositional defiant disorder/conduct disorder

A 14-year-old boy has been diagnosed with CD based on a history of aggression, destruction of property, and deceitfulness. Which evidence-based treatment is likely most cost effective for treatment of his condition? A) Parent training B) Intensive outpatient program (IOP) C) Incarceration D) Military school E) Corporal punishment

Parent training

A 17-year-old young woman is being seen in the office for difficulty with coping at school. She confides that she has problems with eating foods. In trying to distinguish the eating disorders, which of the following is most accurate regarding the differences between bulimia and anorexia nervosa? A) Patients with bulimia tend to be low achievers in academics compared to patients with anorexia B) Patients with bulimia may not have any symptoms until early adulthood, while anorexia typically begins in early adolescence C) Patients with bulimia are less likely to abuse alcohol and have less emotional liability than patients with anorexia D) Patients with bulimia are more resistant to receiving help and often must be forced to see a therapist

Patients with bulimia may not have any symptoms until early adulthood, while anorexia typically begins in early adolescence

A 33-year-old male gymnastics teacher insists that all his students take a shower after class. He supervises the children showering and becomes sexually aroused.

Pedophilic disorder

A paraphilic disorder in which a person has repeated and intense sexual urges or fantasies about watching, touching, or engaging in sexual acts with children, and either acts on these urges or experiences clinically significant distress or impairment

Pedophilic disorder

An 11-year-old boy is brought into the office due to the parents being frustrated with the patient's behavior. The parents report that their son does not want to go to school, doesn't want to study, and becomes angry at small issues. He is found to be deceitful at home and at school. His teachers state that he bullies other children and has stolen other children's property. His physical examination is normal. He is alert and has normal cognition, memory, speech, and language skills. He has no issues with calculations, concentration, or mood. Which of the following medications is most likely to be useful in treating this patient's condition? A) Haloperidol B) Propranolol C) Methylphenidate D) Risperidone E) Lurasidone

Risperidone

Which medication has been shown to be effective for serious behavioral problems, such as aggression, tantrums, and self-injurious behaviors, which may be symptoms of autistic spectrum disorders? A) Fluoxetine B) Risperidone C) Haldol D) Seroquel E) Methylphenidate

Risperidone

A 10-year-old boy presents with episodes of somatic complaints, anxiety, and crying at school which resolves when he is sent home. He won't go anywhere without his mother. Which of the interventions would be an appropriate part of the plan of treatment? A) Place on homebound tutoring to be provided by the school district B) Prescribe lorazepam as needed (prn) for anxiety episodes C) Place the patient on a low dose of fluoxetine D) Immediately restrict access to the mother until anxiety symptoms cease E) Reassure the mother that the patient is going through "a phase" and that this will pass with little impact on the child's subsequent life

Place the patient on a low dose of fluoxetine

Which of the following scenarios is most consistent with factitious disorder? A) Feigning psychosis to avoid criminal charges B) Lying about back pain to receive time off from work C) Psychogenic non epileptic seizures in the context of family conflict D) Placing feces in urine to receive treatment for a urinary tract infection E) Recurrent fears of having a serious illness

Placing feces in urine to receive treatment for a urinary tract infection

Which of the following is a poor prognostic indicator in the treatment of fetishists? A) A stable adult relationship B) Presence of another paraphilia C) Normal intelligence D) Self-referral for treatment E) History of sexual relations without the paraphilia

Presence of another paraphilia

Which of the following initial strategies by the PCP would be the most effective in treating a patient with somatic symptom disorder? A) Antianxiety medication B) Extensive medical workups to provide reassurance C) Referral for psychotherapy D) Regularly scheduled appointments with reassurance E) An antipsychotic medication

Regularly scheduled appointments with reassurance

A 23-year-old man comes to his clinician asking for sexual reassignment surgery. He states that for "as long as I can remember," he has felt that he was born in the wrong body. He states that he believes that "truly I am a woman" and is disgusted by his male body habitus. He has been living as a woman since he moved out of his parents' house several years ago. He wishes to have his penis removed and would like female breasts and genitalia. He considers himself a heterosexual because he is attracted to men. When talking with the patient, which of the following should be used to describe this man? A) He has a paraphilia disorder B) He has fetishistic disorder C) He has gender dysphoria D) She has gender dysphoria E) He is a sadist

She has gender dysphoria

A 42-year-old woman describes a 20-year history of numerous physical complaints, including joint pain, dysuria, headaches, chest pain, nausea, vomiting, irregular menses, and double vision. Although they do not all occur at the same time, she has been suffering from one or more of these problems throughout her adult life. Many medical evaluations have been performed, and she has undergone repeated hospitalizations, but no specific cause has yet been found. Discussion with her husband confirms no traumatic events in the patient's life. She is extremely anxious and has become significantly disabled as a result. Which of the following is the most likely diagnosis? A) Body dysmorphic disorder B) Illness anxiety disorder C) Somatic symptom disorder with predominant pain D) Somatic symptoms disorder E) Conversion disorder

Somatic symptom disorder

A 53-year-old man pulls a back muscle while doing his usual exercise routine 6 months ago. He sought out several medical opinions with sports medicine, orthopedics, and neurology, all of which assured him that he only experienced an unfortunate muscle strain. However, the patient currently exhibits high levels of anxiety and ascribes any ache or pain to some unknown malady. He is so fearful of reinjury that he no longer exercises, always walks with a cane, and is insistent that someone be with him at all times in case he becomes debilitated. Which of the following best describes the situation? A) Factitious disorder B) Malingering C) Conversion disorder D) Somatic symptom disorder E) Illness anxiety disorder

Somatic symptom disorder

A 63-year-old woman returns to her family care provider with continuing headaches for 9 months. She describes the pain as "constant ... always with me," around her entire scalp. She does not appreciate much variation throughout the day, and she cannot name any aggravating or alleviating factors. Although she occasionally feels lightheaded when in severe pain, she denies photophobia, visual changes, nausea, or vomiting. She is especially upset about the headaches, as she retired in the past year and has been unable to travel to visit her infant granddaughter. Complete neurologic examination, computed tomography, magnetic resonance imaging, laboratory studies, and lumbar punctures have been unremarkable. Which of the following is the most likely diagnosis? A) Factitious disorder B) Conversion disorder (functional neurological symptom disorder) C) Illness anxiety disorder D) Malingering E) Somatic symptom disorder with predominant pain

Somatic symptom disorder with predominant pain

A 4-year-old boy who is an only child starts preschool. His parents are quite nervous about this and describe themselves as protective and overinvolved. However, they are looking forward to seeing him more involved with school, as he has had little peer interaction prior to this because he has never shown much of an interest in interacting with others. He has never used many words, which the parents attribute to his isolation. They tell the teacher he has always done best with a strict schedule and doesn't tolerate change well. He has always seemed to have a very narrow repertory of play—focusing primarily on spinning objects such as tops and balls. He comes to the classroom for the first time, runs right to these toys, and does not say goodbye to his mother or even acknowledge she is leaving. Other children attempt to play with him, but his response is to either ignore them or get angry at their advances. His focus on spinning objects in play might be considered what type of behavior often seen with ASD? A) Rigidity B) Stereotyped behavior C) Lack of social reciprocity D) Poor language development E) Obsessional thinking

Stereotyped behavior

A 6-year-old boy with an early, ongoing history of distractibility, hyperactivity, and impulsivity is diagnosed with ADHD. He is treated with methylphenidate. Three weeks later he is brought in, and his inattention and hyperactivity are much better. The mother also notes that he has a small bald spot from where he has begun repeatedly rubbing his head. You periodically observe him to suddenly raise his hand to the spot, rub back and forth once, and put his hand down. There is no rash but the area is hairless. The most likely diagnosis is which of the following? A) Alopecia secondary to sitmulant B) Drug allergy with contact dermatitis C) Stimulant-induced complex motor tic D) Scabies E) Attention-seeking behavior

Stimulant-induced complex motor tic

Patients with schizophrenia or other psychotic disorders may present with delusional claims of cross-gender issues. Which of the following pieces of history would suggest that a male patient's claim of cross-gender identity is due to delusions? A) The patient says he feels as if he is a member of the other sex but does not believe that he is a member of the other sex B) The patient has felt that he was the wrong gender from a young age C) The patient dresses in the attire of the other sex D) The patient actually believes he is a member of the other sex

The patient actually believes he is a member of the other sex

A married pharmacist comes in for treatment at the insistence of his wife, who was disturbed to find that he was wearing some of her undergarments under his clothes. He admitted to her that he often masturbates when wearing her underwear and fantasizes about wearing it while having intercourse with her. Which of the following best describes this paraphilia? A) Fetishistic disorder B) Fetish behavior C) Transvestic disorder D) Transvestic behavior E) Masochism

Transvestic behavior

A 35-year-old man being seen for major depression shares that he enjoys dressing as a woman and masturbating in private. He finds cross-dressing very arousing sexually but is married, and his wife has become aware of this. She is very upset, and there have been marital problems over his behavior. At work and in other settings, he functions in typical male roles and activities. He had two sexual experiences with men before he got married. He feels very committed to his marriage and finds his wife sexually attractive. The best diagnosis for this patient would be which of the following? A) Mixed personality disorder with schizotypal and borderline features B) Gender dysphoria C) Transvestic fetishism D) No diagnosis E) Body dysmorphic disorder

Transvestic fetishism

Which of the following is the most useful approach for the patient in Question 29.1 (grandmother with somatic symptom disorder)? A) Confrontation regarding the psychological nature of her pain B) Prescription for non-narcotic pain medication C) Reassurance that there is no evidence of pain D) Referral to a mental health professional E) Validation of her experience of pain

Validation off her experience of pain


Ensembles d'études connexes

Human A & P II Lecture: Chapter 16

View Set

Chapter 7 - Axial and Appendicular

View Set

Matter and Energy on Earth - Bio

View Set

Solving Linear Systems by Substitution or Graphing

View Set

NOS 130 Lesson 4. Managing the Windows 10 Environment

View Set

MacMillian Questions Chapter 10, 12, and 14

View Set

Psychology Chapter 11 Motivation and Emotion

View Set

acct 1160 test 3 practice questions

View Set

NCLEX QUESTION ADULT HEALTH W/ RATIONALES

View Set